Космос и астрономия

Ответить в тред Ответить в тред
Check this out!
Тред тупых вопросов №135 Hertzsprung–Russell edition Аноним 29/07/20 Срд 06:23:18 5911001
6a01b8d0788cce9[...].png 358Кб, 896x660
896x660
I08-01-HRdiagra[...].jpg 242Кб, 1925x1449
1925x1449
stellarevolution.jpg 104Кб, 750x609
750x609
A.mp4 6746Кб, 1280x720, 00:00:28
1280x720
Тред вопросов о жизни, Вселенной и всём таком.

Спрашиваем то, за что в других местах выдают путёвку в биореактор. Здесь анонимные учёные мирового уровня критически рассмотрят любые гениальные идеи и нарисованные в Paint схемы.

Предыдущий тут: >>588587 (OP)
https://2ch.hk/spc/res/588587.html

Q: Можно быстрее?
A: Можно упасть в пузырь Альбукерке, NASA уже почти надула его.

Q: Я начитался охуительных историй про уфологию, че делать, нам жопа?
A: Да, тебе жопа, можешь сгонять в зогач или куда оттуда пошлют.

Q: Что будет с человеком в вакууме без скафандра / если он упадет на черную дыру / попробует ступить на поверхность газового гиганта/солнца?
A: Он умрёт.

Q: Почему бы не привязать ракету к воздушному шару или стартовать с горы?
A: Космос - это не как высоко, а как быстро, большая часть энергии ракеты уходит на разгон вбок.
Подробнее тут https://what-if.xkcd.com/58/ (английский) https://chtoes.li/orbital-speed/ (перевод)
Аноним 29/07/20 Срд 09:05:11 5911052
>>591097 →
Разгоняюсь в сторону Марса до 0.9с, по моим часам сигнал идет уже не 6-44 минуты, а намного меньше. Где твоя однородность времени теперь?
Аноним 29/07/20 Срд 10:08:02 5911073
>>591100 (OP)
Ебанный стыд...
Во-первых, Алькубьерре.
Во-вторых, не упасть, а создавать вокруг корабля изнутри (иначе кина не будет).
В-третьих, НАСА искривляет пространство на десятимиллионную часть, контролируя это сверхточными интерферометрами, до самого варп-привода здесь - как до Антарктиды раком.
Аноним 29/07/20 Срд 12:55:49 5911224
>>591097 →
Твой чувак на марсе появляется в момент, когда ты слушаешь его ответ, а потом исчезает. А луна исчезает, если на неё никто не смотрит. Стоп, это в outer wilds было.
Аноним 29/07/20 Срд 14:45:34 5911275
>>591105
>Разгоняюсь в сторону Марса до 0.9с
В маняфантазии своей, человек-фотон.
Ирл ты разгоняешься в сторону Марса до 0.000037с и летишь в пространственно-временных координатах на общих основаниях, печально онанируя на лоренц-фактор частицы Oh-my-god.
Аноним 29/07/20 Срд 15:15:53 5911336
>>591127
Да похуй на то, что там ИРЛ происходит, мы о математике говорим и о том, что в принципе возможно. А в принципе возможно заставить с точки зрения наблюдателя заставить события происходить с любой скоростью, а разнесенные в пространстве события происходить в любой последовательности, причем никаких объективных критериев, по которым одну систему отсчета можно предпочесть другой — не существует.
Аноним 29/07/20 Срд 15:19:27 5911347
>>591133
Система отсчета, связанная с реликтовым излучением, нам с начала начал положена. На том стоит и стоять будет Земля!
Аноним 29/07/20 Срд 15:40:39 5911388
>>591134
Относительно реликтового микроволнового фона Земля тоже не стоит, а движется со скоростью ~370 км/с (а Местная группа галактик целиком — аж на 680 км/с), и не с начала начал он нам положен, а с примерно 380 тысяч лет после Большого Взрыва.

Интересно было бы реликтовый нейтринный фон померять и его анизотропию — он расцепился с остальной материей всего через секунду после БВ, так что там никаких крупномасштабных неоднородностей и течений еще не успело возникнуть.
Аноним 29/07/20 Срд 17:48:13 5911509
>>591138
>не с начала начал он нам положен, а с примерно 380 тысяч лет после Большого Взрыва
Тащемта похуй (скорее всего). Чтобы реликтовый фон поехал в какую-то одну сторону, то есть чтобы эта система отсчета была заметно смещена относительно "системы отсчета всего-всего", что-то другое должно было дружно поехать в другую сторону. В это слабо верится как-то.
Аноним 29/07/20 Срд 18:03:32 59115210
>>591133
Ёбаный насрать, если говорить о математике, то я вот прямо сейчас нарекаю себя коловратом тахионом, и все ваши системы отсчёта к чёртовой бабушке летят.

Изначально речь шла про >>591066 → взрыв бетельгейзе, про который пережравший грибов >>591067 →-анон ответил, что "если мы увидели это сейчас, значит это случилось сейчас". А вот нихуя! Ни мы, ни бетельгейзе до заметных лоренц-факторов не разгонялись, поэтому все эти сраные фокусы в нашем случае не работают. И даже 370 км/с относительно реликтового фона это хуйня.
Аноним 29/07/20 Срд 18:16:20 59115311
>>591150
>что-то другое должно было дружно поехать в другую сторону
Так ведь поехало же! Крыша.
Аноним 29/07/20 Срд 21:05:13 59116712
>>591100 (OP)
Интерферометры позволяют увеличить разрешение только по оси между телескопами или во всех направлениях?
Аноним 29/07/20 Срд 23:11:56 59119213
>>591167
Только по оси. С двумя плечами приходится изъебываться, крутя одно из них по орбите или оба друг вокруг друга. Но вообще самокалибрующаяся схема РСДБ предполагает минимум три инструмента.
Аноним 30/07/20 Чтв 01:14:07 59120514
Аноним 30/07/20 Чтв 05:13:38 59121215
wojak-lighting-[...].jpg 83Кб, 750x1000
750x1000
>>591205
>about one million years’ time

Там неопределенность больше, чем вся орбита Земли.
Аноним 30/07/20 Чтв 09:18:06 59121916
NASA-project-or[...].jpg 949Кб, 3000x2400
3000x2400
9.jpg 67Кб, 768x432
768x432
Космоананасы, вот такая мысль. Есть у проект атомно-импульсного корабля, есть проект солнечного парусника. А разве нельзя построить солнечный парусник, который будет разгоняться лазером с накачкой от ядерного взрыва. Это ведь проще, чем классический взрыволёт.
Аноним 30/07/20 Чтв 14:57:09 59125317
>>591219
>Это ведь проще
Ваще как два пальца обоссать.
Аноним 30/07/20 Чтв 19:33:22 59133418
>>591219
Лазер с ядерной накачкой - одноразовый, тебе придётся их расходовать как рабочее тело, чтобы пукнуть световым импульсом с жалкой тягой на единицу массы.
Аноним 30/07/20 Чтв 21:10:29 59135719
Вчера запустили новый марсоход с вертолетиком, который должен прилететь только в конце февраля. Хули так долго? Вроде как огурцов планируют за 3 месяца доставить когда нибудь. А ещё оптимальное окно, ко-ко-ко, раз в два года.
Аноним 30/07/20 Чтв 21:14:20 59136020
>>591334
Так у взрыволёта точно такая же проблема
Аноним 30/07/20 Чтв 21:32:00 59136621
>>591357
>Хули так долго
Новые Горизонты смотрят осуждающе.
Полгода для космоса у него долго.
Аноним 30/07/20 Чтв 22:49:59 59139422
b179ffca6a5fbd4[...].jpg 164Кб, 900x600
900x600
На поверхности луны Европы использовать лёд как строительный материал? Типа строить ледяные замки и они никогда не будут таять, будут стоять там вечно?
Аноним 30/07/20 Чтв 22:54:45 59139923
c2c9c0b730a0933[...].jpg 99Кб, 800x800
800x800
>>591394
Даже на Земле можно.
Аноним 30/07/20 Чтв 23:06:41 59141324
>>591399
А на Земле есть древние ледяные скульптуры? Типа например построили ледяную скульптуру 20 лет назад и она всё еще стоит? Вроде везде достигаются температуры таяния раз в год или несколько лет.
Аноним 30/07/20 Чтв 23:06:43 59141425
>>591399
Как в таком какоть?
Аноним 30/07/20 Чтв 23:09:37 59142026
>>591413
>Вроде везде
В центре Антарктиды точно нет. Какие-нибудь пещеры еще подойдут, наверняка. Но лед один хуй не вечен, он медленно течет, как стекло, например. За сотни-тысячи лет даже при постоянной минусовой температуре статуе пиздец, думаю.
Аноним 30/07/20 Чтв 23:19:25 59143327
>>591414
Удобства во дворе.
Аноним 30/07/20 Чтв 23:19:53 59143428
>>591420
Там наверное будет работать эрозия от ветра В антарктиде.
На Европе может от радиации прилетеющих частиц будет аморфироваться структура, но если сделать огромную статую так чтобы из космоса видно, она долго простоит.
Аноним 30/07/20 Чтв 23:26:09 59143829
banggoes.jpg 368Кб, 700x395
700x395
article-0-0B69D[...].jpg 48Кб, 634x359
634x359
>>591394
На поверхности Европы глубокий вакуум, там даже лёд сублимируется. Но будет сублимироваться долго, если слой толстый, так что норм. Главное чтобы не как у тебя на пике, обливать водой - ибо в вакууме она испарится все равно. Лучше всего лёд бурить, а не строить всякие иглу. Правда непонятно куда стравливать тепло в долгосрочной перспективе, чтоб не таяло.

А вообще изо льда можно даже корабли делать. И делали, точнее из смеси льда и древесной пульпы. Причём даже авианосцы.
https://en.wikipedia.org/wiki/Project_Habakkuk
Аноним 30/07/20 Чтв 23:31:54 59143930
>>591438
>пик
Как они вот так умудрились говно утопить?
Аноним 31/07/20 Птн 01:07:35 59146431
>>591357
Дело не в окне, а в траектории и количестве дельты.
Аноним 31/07/20 Птн 01:35:58 59146932
>>591357
Все вопросы к Вальтеру Гоману. Это этот зловредный инженеришка придумал летать до Марса по полгода.
Аноним 31/07/20 Птн 05:16:21 59148433
Какой будет сперма на вкус в космосе?
Аноним 31/07/20 Птн 05:52:35 59148634
Waterphasediagr[...].gif 123Кб, 1844x1246
1844x1246
>>591438
>На поверхности Европы глубокий вакуум, там даже лёд сублимируется.
Нет. На Европе максимальная температура 125К. При такой температуре лед не сублимируется при любом давлении.
Аноним 31/07/20 Птн 12:06:59 59151435
>>591100 (OP)
Возможна ли галактическая цивилизация типа Империум? Там ещё вроде какие то типы этих цивилизаций есть, так вот, к какому типу она относится?
Аноним 31/07/20 Птн 12:07:57 59151536
>>591100 (OP)
Чем нейтрино отличается от фотона?
Аноним 31/07/20 Птн 12:37:07 59152437
>>591515
Примерно тем же, чем слон от коврика.
Аноним 31/07/20 Птн 13:02:29 59152738
>>591486
Там может еще химия какая происходит, она же красная местами. Вон у Луны есть така хуйня, что реголит поднимается на терминаторе и шагает с рассветом. На Европе всяко что-то подобное может быть, плюс всякие гейзеры. Гейзерное говно на безатмосферных телах еще пизже ебошит, типа частица выходит на суборбитальную траекторию может и потом въебашивается обратно в поверхность.
Аноним 31/07/20 Птн 14:06:18 59154039
>>591524
Спасибо за ответ. Но хотелось бы подробнее.
Аноним 31/07/20 Птн 14:19:32 59154240
>>591540
Примерно всем — участием в фундаментальных взаимодействиях, массой, спином, количеством ароматов, наличием античастиц, и т.д., и т.п., че это за вопрос вообще такой?
Аноним 31/07/20 Птн 14:32:51 59154541
1586275995730.png 29Кб, 741x568
741x568
Так, примерно, можете почувствовать насколько сильно продвинется освоение космоса, если на это будут направленны все допустимые силы и ресурсы человечества? Или текущего финансирования более, чем достаточно и остается только ждать пока моченые придумают нам прыжковые варп-гипердвигатели?
Братишка Аноним 31/07/20 Птн 14:39:08 59154742
Аноним 31/07/20 Птн 14:47:12 59155043
>>591542
Просто говорят, что это это самая быстрая и "всепроникающая" частица, но ведь фотон и так свет, а ничего быстрее света нет.
Аноним 31/07/20 Птн 15:06:00 59155744
>>591545
>если на это будут направленны все допустимые силы и ресурсы человечества?

Человечество просто надорвётся, случится голод, эпидемия, бунт, война и твой космос начнут все ненавидеть.
Вот представь что ты Васян с завода, ты проснулся в 6 утра, пошел на завод через час и пахал там весь день, приходишь в 8 часов вечера домой и просто хочешь вырубиться и спать. Ты думаешь Васяну интересен твой космос, твои варп двигатели, пиу-пиу лазеры на марсе? Ему лишь бы смену отпахать, у него времени на себя и детей не остается. Какая ему польза от космоса в ежедневной жизни? Как он с космосом сознательно взаимодействует, какая ближайшая точка его сознательного соприкосновения с космосом так чтобы ему напомнилось, что этот космос вообще существует? Никак, он о космосе может раз в год думает.
А тут приходишь и говоришь ему, что нужно урезать социалочку, доплаты за многодетность, увеличить срок выхода на пенсию, что он должен работать еще лишние 3 часа в день и всё лишь для того чтобы продвинуть какое-то освоение космоса, который ему в хуй не сдулся. Ты пойми, сынок, ТЕБЕ космос интересен потому что у тебя наверное слишком дохуя времени, потому что за тебя мамка работает. Если бы ты сам стал работать и пахать на заводе, тебе бы времени для всей этой фантастической космической эскапации просто не осталось бы.
Аноним 31/07/20 Птн 16:49:54 59157545
Какой величины должен быть нейтринный телескоп дабы сфоткать панораму реликтовых нейтрино?
Аноним 31/07/20 Птн 17:55:40 59158246
>>591575
Пока что никакой, т.к. с самой регистрацией нейтрино таких малых энергий у ученых большая проблема, не говоря уж о какой-то панораме.
Аноним 31/07/20 Птн 17:58:18 59158347
>>591100 (OP)
Если пердану в космосе, то полечу?
Аноним 31/07/20 Птн 21:32:55 59162048
>>591583
И так летишь. Если без скафандра, то ускоришься. В нем — неа
Аноним 01/08/20 Суб 05:44:04 59165749
bamp
Аноним 01/08/20 Суб 07:01:38 59166050
Если стрельну спермой в космосе, я стану космическим аппаратом?
Аноним 01/08/20 Суб 07:23:15 59166251
БВ произошел одновременно везде, и вселенная расширяется соответственно одновременно везде.
Конечно ли количество материи (обычной барионной) в космосе? Если так, то возможно ли существование планеты, на которой одна сторона неба звездная, другая темная (край вселенной)?
Аноним 01/08/20 Суб 08:57:51 59166752
>>591662
>Конечно ли количество материи (обычной барионной) в космосе

У нас в Нобелевском комитете за такие вопросы сразу убивают нахуй, т.к. это непроверяемая гипотеза. Нам доступна только видимая часть Вселенной, а что там на большем расстоянии происходит — узнать невозможно, принцип причинности запрещает.
Аноним 01/08/20 Суб 12:46:29 59168153
>>591667
Но в видимой вселенной могут быть частицы, запутанные с частицами за горизонтом србытий. Тут может быть способ измерить всю вселенную, а не только видимую.
Аноним 01/08/20 Суб 15:30:22 59170254
1. Можно ли спрыгнуть с парашютом с высоты 400 км? а с 80 км?
Не с орбиты, а просто из космоса с нулевой орбитальной скоростью.
2. Если все же нет, то какая защита потребуется для этого? Меньше, чем у орбитальной банки или нет?
Аноним 01/08/20 Суб 15:56:25 59170855
>>591702
Если ты про обычный затяжной прыжок в высотном костюме/скафандре, и раскрытие купольного внизу на дозвуковой скорости (где он только и работает), то нельзя ни с 400, ни с 80. Перегрузки и нагрев будут слишком большие, нужна жёсткая фиксация спины как минимум, ориентация спиной вперёд, активное охлаждение. Уже у Баумгартнера на 39км перегрузки были основной опасностью, к которой готовились, а Юстас прыгал с 41км в жестком скафандре уже (правда он не был так подготовлен).

Но если не прямо "традиционным" способом, то можно, только не с купольным парашютом, а с надувным коническим. С таким в принципе можно и с орбиты войти (см. PARACONE, НТУ и прочие подобные), но будет сложно управлять и приземлишься хуй знает где. Это всё в теории, естественно, ИРЛ сложности можно понять лишь реализовав. Но по крайней мере в некотором приближении цифры сходятся хорошо.
Аноним 01/08/20 Суб 16:01:41 59170956
>>591681
Запутанность всё равно предполагает передачу взаимодействия, а если её нет то и измерить ты ничего не сможешь
Аноним 01/08/20 Суб 17:07:48 59172557
Где узнать про траекторию эволюции красных карланов?
Аноним 01/08/20 Суб 19:02:56 59173858
Я так понял нейтринная астрономия это найти 10 штук нейтрино раз в пол года?
Аноним 01/08/20 Суб 19:22:40 59173959
>>591738
>найти 10 штук нейтрино раз в пол года
Это охуеть успешный год.
Найти одно нейтрино в год уже заебком.
Но информация от нейтрино уникальна.
Аноним 01/08/20 Суб 19:53:54 59174160
>>591725
Могу только помочь с коричневыми карланами. Чтобы проследить ихнюю эволюцию, посмотри стримы Ежи Сармата
Аноним 01/08/20 Суб 21:35:07 59175461
>>591741
Интересуют только красный и евоная эволюция.
Аноним 01/08/20 Суб 21:38:55 59175562
1558241290030.jpg 19Кб, 946x755
946x755
Сейчас наблюдаю что-то подобное в ночном небе. Почти полная Луна и единственная яркая точка возле нее. Можете сказать что это за звезда или планета?
Аноним 01/08/20 Суб 21:40:02 59175663
>>591755
Юпитер. Еще левее и чуть выше — Сатурн.
Аноним 01/08/20 Суб 21:46:01 59175764
1463224514375.jpg 1261Кб, 1500x1100
1500x1100
Аноним 01/08/20 Суб 21:48:18 59175865
>>591739
Какая блять информация от 1 нейтрино?
Аноним 01/08/20 Суб 21:52:22 59176066
>>591725
Никто 100% не знает, т.к. красным карликам для выгорания требуются триллионы лет, а Вселенной всего 14 миллиардов.

Скорее всего, они просто вырабатывают весь водород, а горение гелия так и не начинается, а т.к. маломассивные красные карлики не имеют ядра (вся звезда состоит из одной конвективной зоны), то они просто тухнут без каких-либо спецэффектов и сжимаются в белый карлик.
Аноним 01/08/20 Суб 22:10:29 59176167
>>591758
Энергия, вектор прихода, заряд, может спин, хер еще знает, может он при столкновении распадается на еще какие-то частицы и эти частицы также интересны.
Аноним 01/08/20 Суб 22:25:57 59176268
>>591758
По направлению и силе события можно понять откуда прилетело, и совместить с другими инструментами.
Аноним 01/08/20 Суб 22:32:56 59176369
1323799135059.jpg 31Кб, 321x321
321x321
Аноним 01/08/20 Суб 22:46:44 59176570
>>591763
Вообще-то да, заряд. Лептонный у самого нейтрино, и электрический у продуктов реакции — антинейтрино и нейтрино произведут позитрон и электрон соответственно при взаимодействии с водой в детекторе.
Аноним 01/08/20 Суб 23:02:12 59176671
Аноним 01/08/20 Суб 23:19:31 59176872
>>591765
Лептонное число ты только на основе продуктов жизнедеятельности можешь предположить.
Аноним 02/08/20 Вск 02:04:49 59180273
Посмотрел в спейселжин на центр нашей любимой родной галактики и увидел там прям дохулиард звезд в одном месте. От центральной сверхмассивной ЧД некоторые находились на расстоянии всего в несколько а.е.
Вопрос, насколько тепло в центре млечного пути? Теплее ли там, чем в вакууме между солнцем и альфа центаврой?
Аноним 02/08/20 Вск 03:23:44 59181274
Какие заряды в принципе есть у частиц?
Аноним 02/08/20 Вск 12:35:22 59186275
157820788915782[...].jpg 15Кб, 730x410
730x410
Аноним 02/08/20 Вск 13:48:17 59187476
Аноним 02/08/20 Вск 14:54:32 59189677
>>591862
Явно не HTPB/APCP, на жижу смахивает. По рыжеватому дымку можно подумать на несгоревший избыток азотного тетраоксида, разложившегося в двуокись. Но факел на гидразиновый тоже не похож, горючкой там может быть хз что. Непонятно только нахуя это всё в военной мелкоракете, правда.
Аноним 02/08/20 Вск 15:10:43 59191378
>>591896
По-моему, это пыль просто желтая, а не дым.
Аноним 02/08/20 Вск 17:18:31 59194779
274px-Meateater[...].jpg 12Кб, 274x183
274x183
Если уменьшить человека до размера муравья, сможет ли он видеть? Ведь колбочки и палочки рассчитаны на определенную длину волны, разве нет? Или он просто начнет видеть в другом спектре?
Аноним 02/08/20 Вск 17:29:36 59194880
>>591947
уменьшающих лучей, излучения и капсул не существует, твой вопрос также бессмыленен, как если бы ты спросил "Сможет ли заклинание Вингардиум Левиоса захватить воздух в комнате и перенести его в другую комнату, создав в первой комнате вакуум?"
Аноним 02/08/20 Вск 17:50:04 59194981
.jpg 65Кб, 227x1043
227x1043
>>591947
"Уменьшать в том виде в котором есть" - бессмысленно, как сказал анон выше.

Если вывести породу микрочеликов размером с муравья, видеть они смогут конечно. Но для них дело будет не в колбочках, а в размере апертуры. Чем больше собирающая поверхность, тем чувствительней оптическая система. Чем меньше, тем меньше. Разрешение тоже меньше будет, скорее всего. Также будет меньше разнесение глаз, бинокулярное зрение хуже.

В целом чем больше глаз, тем лучше у него потенциал для острого зрения. Но 1) гарантии нет - это уж как эволюция решит и 2) для того чтобы разбирать хайрез картинку, надо еще и хайрез мозги иметь, так-то.
Аноним 02/08/20 Вск 17:52:38 59195182
>>591949
А если собрать из специальных мелких атомов?
Аноним 02/08/20 Вск 17:54:11 59195283
>>591951
Вот потому и бессмысленно, что таких нет
Аноним 02/08/20 Вск 17:55:06 59195384
d9597e005685818[...].jpg 20Кб, 494x484
494x484
Аноним 02/08/20 Вск 17:58:20 59195485
>>591948
Сможет, инфа сотка
мимомаксвелл
Аноним 02/08/20 Вск 20:51:55 59200686
>>591100 (OP)
Каково было бы жить "по соседству" со звездой вроде Бетельгейзе? Эта хрень в диаметре больше Солнца в 950-1200 раз. Как бы это выглядело на небосводе, находись она на месте Альфа Центавры?..
Аноним 02/08/20 Вск 21:07:26 59203087
>>592006
Как точка. И в 10000 раз больше — как точка. Человеческий глаз не может различить ничего меньше 1 угловой минуты, а угловой диаметр Альфы Центавры А — 7 тысячных угловой секунды. На расстоянии в несколько световых лет радиус звезды должен быть как минимум в 50-100 тысяч раз больше солнечного, чтобы её было видно как маленький диск, а не точку, но таких больших звезд не бывает.
Аноним 02/08/20 Вск 21:10:18 59203688
>>592030
Но светимость-то больше (в 100к от солнечной раз примерно). На ночном небе должна выделяться
Аноним 02/08/20 Вск 21:10:44 59203989
>>592006
>Бетельгейзе
>на месте Альфа Центавры
Звездная величина уменьшится единиц на 10 по сравнению с Альфой Центавра сейчас. Будет просто яркая точка уровня Венеры, а то и просто яркая звезда.

Чтобы ебический диаметр был заметен, ей нужно быть намного ближе, в двойную систему к Солнцу желательно.
Аноним 02/08/20 Вск 21:12:32 59204890
>>592036
На небе много чего выделяется, но это всё равно будет точка, пусть и довольно яркая.
Аноним 03/08/20 Пнд 22:06:15 59276391
Почему у Марса вся южная часть в кратерах, но на северном полушарии кратеров практически нет?
Аноним 03/08/20 Пнд 22:30:31 59276892
>>591100 (OP)
Красные карланы почему белыми становятся? Я думол что они тухнут просто.
Аноним 03/08/20 Пнд 22:53:25 59277793
>>591152
>если мы увидели это сейчас, значит это случилось сейчас". А вот нихуя!
Хуя, чмоня. Не имеет никакого физического смысла пиздеть про то, что это типа в прошлом было. Так может думать только макака экстраполируя свои тупорылые наблюдения на вселенную, когда как на самом деле ничего не может распространяться быстрее скорости света, т.к. это суть реальности.
Аноним 03/08/20 Пнд 22:57:53 59278194
>>591557
Так он сам виноват, что женился и детей заделал, не делал бы бесполезных мясовичков, мог бы с мужиками о космосе общаться.
Блять больше всего я ненавижу мудаков которые говорят , что космос не нужен.
Я всегда ссу на ебало этим уродам, которых к сожалению огромное количество. Сраные ебаные кретины, тупорылое быдло мечтающее о теплом угле, вкусной еде и мясной дырке. Неужели сложно понять простую вещь - Движение вперед это охуенно. Фронтир, неизведанное, полететь на Марс потому что можем и дальше.
И вот эти чмыри которые ненавидят космос, видят только унылое его использование приходят в спейсач и начинают кукарекать, что Марс не нужен.
Сдохните твари. Все что создано вокруг вас руками человека создано наперекор таким как вы. Все время существования человечества мы волоком тащим вас визжащих,упирающихся еблом по говну, камням и грязи вперед к прогрессу и таки протащили из обезьяны к запускающим в космос корабли разумным существам.
Аноним 03/08/20 Пнд 23:05:48 59278495
>>592781
Прогрессор бонбанул.
Аноним 03/08/20 Пнд 23:06:51 59278696
>>592784
В печь этого примата.
Аноним 03/08/20 Пнд 23:31:41 59279497
>>591557
> Человечество просто надорвётся, случится голод, эпидемия, бунт, война и твой космос начнут все ненавидеть.
> Вот представь что ты Василий с деревни, ты проснулся в 4 утра, пошел на поле через час и пахал там весь день, приходишь в 10 часов вечера домой и просто хочешь вырубиться и спать. Ты думаешь Васяну интересен твой автомобиль, твои самолеты, пиу-пиу атомы? Ему лишь бы в поле отпахать, у него времени на себя и детей не остается. Какая ему польза от технологий в ежедневной жизни? Как он с самолетом сознательно взаимодействует, какая ближайшая точка его сознательного соприкосновения с самолетом так чтобы ему напомнилось, что этот самолет вообще существует? Никак, он о железной птице может раз в год думает.
> А тут приходишь и говоришь ему, что нужно работать еще лишние 3 часа в день и всё лишь для того чтобы продвинуть какое-то освоение дальних земель, авиации и автомобиля, который ему в хуй не сдулся. Ты пойми, гимназист, ТЕБЕ прогресс интересен потому что у тебя наверное слишком дохуя времени, потому что за тебя мамка работает. Если бы ты сам стал работать и пахать на поле, тебе бы времени для всей этой фантастической техники просто не осталось бы.
А потом грузовик стал привозить в деревню товары, самолет дал их детям быстрый и безопасный способ перемещения, их внуки смотрели телевизор, питаемый атомной электростанцией, их правнуки капчуют через оптоволоконный кабель и ищут путь в любых ебенях через спутники навигации... А все почему? Потому что кто-то не поскупился развивать это. Космосом занимались с приложением огромных ресурсов целые блять ГОСУДАРСТВА, и благодаря этому ты сейчас сидишь здесь и пишешь хуйню. А если государство снова приложит эти ресурсы, дети Васяна будут контролировать роботизированный космический завод, после чего вернутся в родной Мурманск, освещаемый орбитальным зеркалом и целиком обогреваемый благодаря ТЯЭС на гелии-3.

Но кошмош нинужен, нихуя в жизнь не привнес, это все для мичтателей, ВОТ БЫ ИХ НА ЗАВОД НАХУЙ. Просто в школе васяну слишком хуево разъяснили, что все, что у него есть, когда-то жило на государственные дотации.
Аноним 03/08/20 Пнд 23:36:26 59279698
>>592794
>освещаемый орбитальным зеркалом
На земляшке наоборот надо тень вешать чтобы законтрить глобальное потепление.
А вот для марса зеркала будут очень кстати.
Аноним 03/08/20 Пнд 23:38:27 59279799
>>591557
>Какая ему польза от космоса в ежедневной жизни
Если у Васяна есть вайфай, телефон и навигатор, пользы у него уже дохуя.
Аноним 03/08/20 Пнд 23:39:26 592798100
>>592797
Для всего этого космос не нужен.
Аноним 03/08/20 Пнд 23:40:46 592799101
>>592798
>телефон
>космос нинужен
>навигатор
>космос нинужен
С кем я сижу на одной доске, блядь
Аноним 03/08/20 Пнд 23:42:41 592800102
>>592799
Правильно, съеби отсюда, довен ебаный.
Космос ему для телефона нужен, тупень сраный.
Про наземную навигацию не слышал, чмохен тупорылый.
Аноним 03/08/20 Пнд 23:48:29 592801103
>>592794
>осмосом занимались с приложением огромных ресурсов целые блять ГОСУДАРСТВА, и благодаря этому ты сейчас сидишь здесь и пишешь хуйню.

Государства на мирный космос прикладывали буквально копеечные ресурсы - единственное исключение это сша с их луной. Государства прикладывали большие ресурсы на палки-убивалки. А на твое "а потом будет огого" государствам было насрать как и тому васяну. Не надо путать фантастику/пропаганду с реальностью.

> А если государство снова приложит эти ресурсы, дети Васяна будут контролировать роботизированный космический завод,

Если государства вновь приложат эти ресурсы, никакого космического завода не будет, а будет еще больше палок убивалок.
Аноним 03/08/20 Пнд 23:52:06 592802104
>>592801
Чтобы делать палки-убивалки, нужно технологий, однако. А технологии можно использовать в мирных целях, однако.
>>592800
Зарепортил токсичного долбоеба, пошёл нахуй с доски. В океане как будешь точно ориентироваться, чмо?
Аноним 03/08/20 Пнд 23:54:57 592804105
>>592802
>В океане как будешь точно ориентироваться, чмо?
По звездам, хуесосина ты тупорылая. Для этого космонавтика не нужна, это умели делать тысячелетиями, дегрод.
Срыгнись с доски и не пизди хуйню о которой не знаешь, ты обосрался. Телефоны у него требуют космоса, охуеть, имбецил ебаный.
Аноним 04/08/20 Втр 00:00:27 592805106
>>592804
И тысячелетиями ссали отплывать от берега.
)
Толсто-зелено, долбоеб пришел на доску про космос и начинает визжать, что космос нинужен.
Аноним 04/08/20 Втр 00:03:17 592806107
>>592805
Ты обосрался, обтекай, дегрод.
Без космоса не может в навигацию, охуеть.
Аноним 04/08/20 Втр 00:25:53 592810108
>>592806
Спутниковая съемка для составления точных карт.

Нинужно кста.
Аноним 04/08/20 Втр 00:26:45 592812109
>>592810
Лучшие и самые четкие снимки делаются аэрофотосъемкой.
Аноним 04/08/20 Втр 00:40:04 592818110
>>592812
Покрой регулярной аэрофотосъемкой всю планету.
Аноним 04/08/20 Втр 00:40:28 592819111
Аноним 04/08/20 Втр 02:49:57 592861112
>>592763
Вопрос значительно менее тупой чем кажется, на эту тему не одна сотня хуев сломана, надо шерстить журналы Icarus и изучать текущий консенсус или набор гипотез.
Аноним 04/08/20 Втр 02:50:38 592862113
>>592861
>журналы типа Icarus
фикс
Аноним 04/08/20 Втр 02:52:13 592864114
>>592763
На северном полушарии окиян же был, в окияне кратер так просто не сделаешь.
Аноним 04/08/20 Втр 06:05:26 592894115
EQNMdjbWkAA2EG-.jpg 93Кб, 610x433
610x433
>>591100 (OP)
Вот от ракет падает после полета много всякого ненужного, что говорят сразу раздербанивают на цветмет. А есть возможность где-то обломок этого всякого прикупить дабы положить на полочку.
Я понимаю что желание достать помятую рулевую камеру целиком это перебор, но кусок обшивки бака, сопла или стабилизатора хотелось бы иметь в коллекции. Ну или кусок обгоревшей теплозащиты союза. Ну или что-нибудь другое либо летавшее в космос либо учавствовавшего в выведении.
Аноним 04/08/20 Втр 06:27:08 592896116
>>592894
Можешь сгущенки за шпионаж словить, если вдруг пропалишься.
>учавствовавшего
Или за грамматику.
Аноним 04/08/20 Втр 07:14:18 592899117
>>592896
>Можешь сгущенки за шпионаж словить
А тех, кто на цветмет распиливают тоже сажают? Оно же и так в чистое поле падает. Речь то не о большом куске изделия, а о мелком обломке уровня куска обшивки.
Аноним 04/08/20 Втр 08:21:46 592904118
>>592894
Легче купить бочку гидрозина и окунутся в неё целиком. Или спиздить уран в институте, где есть разрешение на радиоактивные вещества и подсыпать по утру его себе в чай. Эффект примерно такой же.
Мысль понятна? И да за всё это тебя ещё и на сгущёнку посадят.
>>592899
Они этот метал сразу продают людям которые никуда никогда не сядут. А ты будешь хранить вещьдок у себя в квартире.
В целом если у тебя она уже есть, пока квартиру не обыщут тебе ничего не будет. Проблема в том, что когда ты будешь её добывать тебя по горячим следам сразу упекут. На перрон не успеешь дойти.
Аноним 04/08/20 Втр 08:29:29 592905119
>>592904
Ой да ладно.
Это же тайга ебучая.
Кто там ловить-то будет, и как?
мимокрок
Аноним 04/08/20 Втр 08:42:23 592906120
>>592905
Ты не по тайге ходить будешь, а неизбежно выйдешь к дорогам которых там 1 или 2, облегчая контроль. В тайге дебилов как раз легко ловить из-за этого малого количества дорог к которым дебилы липнут.
Аноним 04/08/20 Втр 08:49:26 592908121
>>592904
>гидрозина
Рогозина. На пике очевидная боковушка от Союза.
Аноним 04/08/20 Втр 08:50:19 592909122
>>592904
>Легче купить бочку гидрозина и окунутся в неё целиком. Или спиздить уран в институте, где есть разрешение на радиоактивные вещества и подсыпать по утру его себе в чай.
Ты бы еще водородную бомбу предложил. Охуеть сравнение с кусками железа, свободно валящимися в лес десятками в год. Особенно с предложением окунуться или в чай подсыпать. Это ты утверждаешь, что у союза обшивка баков так же опасна как гидразин с ураном, или ждешь что меня сразу расстреляют без суда за обладание прям в квартире?
Подожду менее истеричного анона.
Аноним 04/08/20 Втр 10:55:09 592921123
>>592909
Я оговорился не гидразин, а гептил. Я к тому, что их не просто так оцепляют и вывозят из леса.
Аноним 04/08/20 Втр 11:06:27 592922124
>>592921
нет там гептила, дурик
только немного перекиси для турбонасоса, но она опасна только если целиком обмажешься ей
Аноним 04/08/20 Втр 11:31:55 592924125
>>592922
А я люблю обмазываться концентрированной перекисью и ААААА
Аноним 04/08/20 Втр 15:22:40 592982126
333.jpg 273Кб, 1024x768
1024x768
>>592781
> ненавижу мудаков которые говорят , что космос не нужен

А где я это говорил? Я лишь констатирую факт, что большинство людей так думает. Вы вот высираетесь мне в ответ, но вы так и не можете провести прямую связь между жизнью Васяна и космосом, вам нужно куда-то в абстракцию лезть, в будущие проекции, в теорию государственной поддержки. Это всё пустое, слишком сложное и далёкое. Васян должен своей кожей этот космос чувствовать, как футбол, как зиму, как налоги, как водку. Вот если бы в космос летали российские десантники космических войск - Васян бы это прочувствовал. Если бы Васян покупал космическую водку произведенную на заводе на орбите - Васян это бы чувствовал. Если бы на 9 Мая над Кремлём происходил не салют, а метеорный дождь из управляемых зондов сводимых с орбиты - Васян это бы чувствовал. Вам нужно находить применения космоса, которые как можно ближе к Васяну, тогда Васяну будет интересен космос и тогда Васян будет что-то жертвовать ради космоса. А до тех пор Космос это как Нарния, Средиземье и мир Гарри Поттера, а Васян это орк-маггл неспособный чувствовать вашу магию космоса.
Аноним 04/08/20 Втр 15:24:32 592983127
image.png 492Кб, 750x600
750x600
Аноним 04/08/20 Втр 15:26:25 592984128
>>592983
Вот пидорасы, я тоже свои посты буду на всяких яплакал и жж-форумах репостить, а потом кидать в вас копипастой.
Аноним 04/08/20 Втр 15:30:39 592985129
Почему радиус шварцшильда равен размеру видимой вселенной?
Аноним 04/08/20 Втр 15:38:07 592987130
>>592985
Хамишь, парниша. Космологический ГС ≠ ГС ЧД.
Радиус Шварцнеггера равен горизонту событий невращающейся черной дыры.
Аноним 04/08/20 Втр 15:52:53 592992131
>>592781
>Так он сам виноват, что женился и детей заделал, не делал бы бесполезных мясовичков, мог бы с мужиками о космосе общаться.
Сорян, без бесполезных мясовичков не будет и всего остального. А вообще это не твоё собачье дело, кто что предпочитает.

>Все что создано вокруг вас руками человека создано наперекор таким как вы.
Всё, на что ты дрочишь в ютуб-роликах на научно-популярную тему создано на налоги таких ржаных-гречневых. Уважай чужой труд, пидор.
>Все время существования человечества мы волоком тащим вас визжащих,упирающихся еблом по говну, камням и грязи вперед к прогрессу и таки протащили из обезьяны к запускающим в космос корабли разумным существам.
Больше чем уткнувшееся в корыто быдло я ненавижу таких восторженных любителей научпопа-быдло хотя бы не шкварит собой вещи, которые я люблю-в том числе космонавтику. Кто вы-то, хуесос? Толпа визжащих о новуке пикейных жилетов?
Аноним 04/08/20 Втр 16:02:11 593002132
Аноним 04/08/20 Втр 16:10:07 593003133
>>592802
>Чтобы делать палки-убивалки, нужно технологий, однако.

Но это никак не противоречит - никаких космических заводов от лозунга "ряяя государство направит средства в космос" не будет.
Так как такого никогда не было.

> А технологии можно использовать в мирных целях, однако.

Подобная модель действовала в короткий исторический период с второй мировой до конца холодной войны. Ну то есть пока была полувоенная мобилизация ресурсов. Сейчас уже лет 20 создание технологий либо финансирует бизнес напрямую, либо тот же бизнес под контролем государства через налоговые вычеты.

Собственно показателен пример VentureStar/x33 - просто слили кучу бабла а потом все дропнули. А лозунг был - счас сделаем на госденьги технологию SSTO а коммерсы ее в мирных целях используют. И так сейчас всё.
Аноним 04/08/20 Втр 16:26:02 593012134
Аноним 04/08/20 Втр 16:26:56 593013135
Аноним 04/08/20 Втр 16:49:57 593014136
Почему пишут "световой год", если "петаметр" короче и примерно того же порядка?
Аноним 04/08/20 Втр 16:51:48 593016137
>>593014
Ты ещё аттопарсек помяни.
Потому что световой год это точно кратная единица, как и парсек, которые удобней каждая в своём случае.
Аноним 04/08/20 Втр 21:46:19 593143138
Как теоретически в принципе взаимодействует нейтрино? Есть ли теоретические способы детектировать реликтовое нейтрино, или это в принципе неизвестно?
Аноним 04/08/20 Втр 22:18:04 593170139
А если на Марсе найти равнину на которой нету никаких преград до горизонта, там будет замете изгиб поверхности? ну типа вы чё на шаре живёте? вы живёте на плоскости
Аноним 04/08/20 Втр 23:12:39 593189140
https://meduza.io/feature/2020/08/04/virgin-galactic-predstavila-proekt-sverhzvukovogo-passazhirskogo-samoleta-on-budetrazvivat-skorost-do-treh-chisel-maha каким образом они хотят разогнать до 3 маха, если самый быстрый самолет блэкберд летал около этого и там применялись всякие суперхитрые технологии, а после каждого полета приходилось чинить обшивку?
Аноним 04/08/20 Втр 23:30:45 593196141
>>593016
А потом возмущаетесь насчет дюймов и миль.
Аноним 04/08/20 Втр 23:35:55 593200142
>>593189
это очередной пиздёж говномидаса бренсона перед какой-то операцией с акциями, он постоянно срёт лапшой с говном всяким даунам, которые это за чистую монету принимают
Аноним 05/08/20 Срд 00:32:12 593209143
>>593189
1. В спейсаче принято давать оригинальные линки на первоисточники, или по крайней мере на специфические аэрокосмические издания, а не на левые мурзилки, потому что они постоянно что-нибудь перевирают, если не специализируются на этом. Вот прямо там есть ссылка на первоисточник.
https://www.virgingalactic.com/articles/virgin-galactic-unveils-mach-3-aircraft-design-for-high-speed-travel-and-signs-memorandum-of-understanding-with-rolls-royce/

2.Не надо делать из дрозда какую-то ёба-священную корову. Дрозд был специфическим самолётом, и был собран микроколлективом на скорую руку из засекреченных говна и палок, по принципу Парето (почитай книжцо Бена Рича Skunkworks, он там такие байки про трейдоффы травит что охуеешь). Тот же мигарь 25 летал на ненамного меньших скоростях, и мог кратковременно разгоняться до его скорости, при этом будучи почти обычным серийным самолётом, и ограничением был ультраговенный ресурс двигла и срыв потока на турбине.
Во-вторых, ещё JFK хотел такую хуйню забабахать, но решили что ДОРАХА
https://en.wikipedia.org/wiki/Lockheed_L-2000
https://en.wikipedia.org/wiki/Boeing_2707
Больше того, Конкорд в восьмидесятых тоже хотели модифицировать до 3 махов и движка с высокой двухконтурностью, как у всех современных лайнеров (он летал по сути на военном двигле с афтербёрнером, жор и шум в т.ч. оттуда), все цифры у них сходились, но уже тогда Конкорд был на грани ненужности, поэтому не стали.

>и там применялись всякие суперхитрые технологии
По меркам шестидесятых.

Я бы больше сомневался в нужности кому-либо такого самолёта, чем в возможности его запилить. Большая часть конкордовских минусов особо никуда не денутся.
Аноним 05/08/20 Срд 00:56:04 593212144
>>593189
X-15 летал быстрее, и че? Спейсшип 1-2 тоже. Это ракетные самолеты, но сути не меняет. Ролс-ройс вполне шарит в скоростных движках, они с ним и партнерятся, чтобы не получилось как в SS2, три раза меняли топливо в своей гибридной еботе и переделывая крафт каждый раз почти заново под это.
Аноним 05/08/20 Срд 03:18:58 593251145
Аноним 05/08/20 Срд 03:38:14 593261146
Аноним 05/08/20 Срд 05:06:30 593271147
>>593196
Это-то что.
Ты вспомни чем астрогномы массу измеряют. ЗЕМЛЯМИ И СОЛНЦАМИ.
Аноним 05/08/20 Срд 06:44:20 593272148
>>593196
Начнём с того, что,будучи либеральными в полном принятии новых систем счисления, мы всё ещё помним Гюйгенс. Не забудем не простим.
Аноним 05/08/20 Срд 06:53:29 593274149
>>593272
А че Гюйгенс? Нормально отработал.
Ты может Марс Клаймет Орбитер имел в виду, который сгорел нахуй из-за пиндосни которая лбф вместо ньютонов юзала?
Аноним 05/08/20 Срд 06:58:30 593276150
>>593274
Насколько я помню, половина фоток Гюйгенса проёбана по похожей причине.
Аноним 05/08/20 Срд 07:03:49 593279151
>>593276
Там не прошла команда на включение второго канала связи на Кассини. Мне сложно представить, как это может быть связано с системой мер.
Аноним 05/08/20 Срд 07:13:29 593284152
Стикер 191Кб, 500x500
500x500
>>593279
Расчеты проводили в джоулях а число подали в фут-фунтах.
Аноним 05/08/20 Срд 08:49:46 593294153
>>593272
>>593276
Ты попутал с Mars Climate Orbiter.

>>593279
Не совсем, команда вообще отсутствовала в полётном задании, и она была не с Земли, а от Кассини к Гюйгенсу, при финальной инициализации перед отделением Гюйгенса (за ~ 2 недели до Титана). При разработке ESA подала в NASA документацию по процедуре инициализации в NASA, в которой была описана среди прочего команда включения второго эталона частоты. Но полётное задание в NASA оказалось без этой команды, и прошло при этом три ёбаных проверки каким-то макаром, в обоих агенствах - две при разработке, одну перед прилётом.
Аноним 05/08/20 Срд 13:37:22 593370154
>>593261
Короче говоря бочка трития на 12 000 км диаметром и мы в шоколаде.
Аноним 05/08/20 Срд 13:38:39 593372155
>>593370
Абажжи, тритий не зажжется нахер? Это водороды с дейтериями надо сильно зажать, а тритий-то помягче
Аноним 05/08/20 Срд 14:29:09 593413156
Аноним 05/08/20 Срд 14:30:53 593415157
>>593413
Ничего страшного, кто-то поумнее ответит.
Аноним 05/08/20 Срд 16:00:11 593444158
>>592006
Свет, влетая в атмосферу, начинает рассеиваться. Некоторые лучики улетают в пизду по случайным направлениям, и какое-то их количество прилетает в твой глаз по кривой траектории — сначала чуть отклонившись вбок, потом отклонившись обратно. Из-за чего звёзды и кажутся тебе больше, чем они есть. Поэтому да, если источник света будет ярче, то и количество таких рандомных фотонов будет больше. Размер звезды тут не при чём совершенно.
Аноним 05/08/20 Срд 16:14:03 593453159
>>591433
Один срет второй отпиливает.
Аноним 05/08/20 Срд 21:06:45 593569160
>>591433
Слышал я такую историю, что так же сел посрать человек, в мороз на крайнем севере и ему потом примёрзшую какулю вырезали.
Аноним 05/08/20 Срд 21:10:29 593570161
Просветите меня пожалуйста, о том какие химические реакции будет перспективно применять в будущем, кроме метан-кислород.
Аноним 05/08/20 Срд 21:17:00 593574162
>>593570
фтор и диборан/пентаборан
Аноним 05/08/20 Срд 21:24:55 593576163
>>593574
Фтор 100% никто не будет использовать из-за лютой токсичности и способности к коррозии его соединений, это мертворожденная тема. Все, кто хотел, уже поэкспериментировали и пришли к выводу, что от фтора гораздо больше проблем, чем пользы.
Аноним 05/08/20 Срд 21:42:22 593577164
Кто-нибудь стрелял спермой в потолок МКС?
Аноним 05/08/20 Срд 21:47:04 593580165
>>593577
а где на мкс потолок?
Аноним 05/08/20 Срд 23:17:31 593614166
>>593570
Нитрат гидроксиламмония (HAN) как долгохранимое топливо на замену гидразину. Умеренно канцерогенное топливо, но все равно намного менее ядовитое чем гидразин, может юзаться людьми в базовой защите без подготовки, что сильно упрощает дело. Плотнее, стабильней, безопасней и немного лучше гидразина по импульсу. Можно наддувать азотом вместо гелия. У насы есть смесь AF-M315E, у китайцев ещё какая-то своя, и т.п. все малотоксичные.

Минусов тоже дохрена:
- катализатор начинает нормально работать при ~400 С, и чем больше расход, тем больше его нужно, тем больше жрёт преднакал и тем сложнее его потом охлаждать, т.е. плохо масштабируется до больших движков. Пытаются в схемы с саморазогревом, пока ещё процесс не до конца опробован на практике.
- жижа довольно вязкая, т.к. это расплавленная соль по сути (ионная жидкость). Число Рейнольдса в малых баках очень низкое, сложно обеспечить равномерное поступление и 100% расходование бака.

Теоретически можно использовать с динитрамидом аммония в качестве второго компонента, пока серьёзных испытаний никто не делал.
Аноним 05/08/20 Срд 23:24:26 593616167
>>593570
Под ISRU это неизбежно будут простые соединения и криогенные температуры, требующие активного охлаждения - водород, кислород, простейшие углеводороды вроде метана. Есть ещё смесь воды и алюминиевого порошка, твёрдое топливо (лёд). Для гибридных движков - сложные полимеры вроде нейлона в виде горючего.

>>593574
Валентин Петрович...
Сразу O2F2 тогда уж
Аноним 06/08/20 Чтв 01:52:21 593632168
1. Есть ли у млечного пути орбита вокруг какой-то мега-галактики? Или все галактики имеют примерно одинаковый вес, поэтому просто болтаются и иногда сталкиваются друг с другом?
2. Как работает пузырь альбукьере? В научпопе говорят, что он сжимает пространство спереди и разжимает сзади, но я не понимаю, как это помогает перемещаться быстрее скорости света. На ум только приходит идея, что он сжимает все необходимое для варпа пространство ну типа от солнца к альфа центавре, по четвертому измерении переходит в паралельное пространство и по нему проходит более короткий путь, а потом обратно возвращается в наше. Но блять, это же безумно пиздец.
3. Вопрос о том же искривлении пространства. Что я буду чувствовать, если пространство вокруг меня как-то искривится? Если оно растянется, разорвет ли меня к хуям? А если начнет сжиматься?
Аноним 06/08/20 Чтв 01:58:29 593634169
1. Млечный путь входит в состав Местной группы галактик и гравитационно связан с галактиками в ней, та в состав Местного листа, тот - Местного сверхскопления, и т.д. Орбита - это не обязательно "вокруг", это просто траектория.
Аноним 06/08/20 Чтв 01:58:51 593635170
Аноним 06/08/20 Чтв 02:30:46 593639171
unnamed (2).jpg 19Кб, 170x255
170x255
>>593616
F2NH3, только хардкор
Аноним 06/08/20 Чтв 03:05:46 593641172
>>593632
2. Зачем по четвёртому измерению? Движется внутри тех же трёх. Просто делает так, чтобы сам путь стал короче. У тебя до сих пор какие-то магические представления о сути подобных убердевайсов. Там есть откуда взяться другому измерению если пространство просто сжимается? Ну и зачем приплетать то, чему нет никаких причин?
3. Зависит, наверное, от скорости. Материя в тебе стянута всякими разными силами, вряд ли с ней так просто можно что-нибудь сделать.
Аноним 06/08/20 Чтв 03:48:23 593643173
>>593632
>пузырь альбукьере
Ёбаный стыд...
Аноним 06/08/20 Чтв 04:09:14 593649174
>>593569
>>593453
Кто срал на морозе в такую хуйню не поверит.
Если бы температура воздуха близкая к абсолютному нулю была не смертельной, можно было бы даже так срать.
У воды огромная теплоемкость, а говно по большей степени вода, так что чтобы оно начало замерзать надо несколько минут сидеть было бы.
Аноним 06/08/20 Чтв 06:02:23 593654175
>>593643
Во-первых, "ёбанный".
Аноним 06/08/20 Чтв 06:45:04 593655176
>>593654
Почему?
Ёбанный - это тот, кого выебали.
Ёбаный - это тоже, но и "плохой". По крайней мере так на вике пишут.
У тебя есть пояснения со ссылками почему так а не иначе?
Аноним 06/08/20 Чтв 06:48:01 593656177
А возможно порвать пространство?
Аноним 06/08/20 Чтв 06:52:59 593657178
>>593655
В альбукеркопасте же так написано, ну. Так что традиция, типа как "оффициальный хуйнянейм тред".
Аноним 06/08/20 Чтв 07:00:00 593658179
>>593657
А, ты об этом. Тогда да, двачую поправку.
Аноним 06/08/20 Чтв 10:44:12 593673180
>>593656
>А возможно порвать пространство?
Если ты не боженька, то нет. Материальным тварям доступно порвать только материю.
Аноним 06/08/20 Чтв 10:56:16 593675181
>>593641
> 2. Зачем по четвёртому измерению? Движется внутри тех же трёх. Просто делает так, чтобы сам путь стал короче.
А как он это делает тогда? Если полетит в сторону сжватого простраства, то и сам сожмется же.
Аноним 06/08/20 Чтв 11:08:03 593676182
>>593675
Если сожмётся вместе с ней то да, а так это как по скомканной скатерти пройтись.
Аноним 06/08/20 Чтв 11:15:53 593677183
>>593676
Покажи как ты комкаешь скатерть в 2d. Как раз на плоском мониторе должно получиться.
Аноним 06/08/20 Чтв 11:30:02 593681184
>>593677
В обратную сторону нежели расширение вселенной. Атомы то никуда не распухли.
Аноним 06/08/20 Чтв 11:59:20 593685185
>>593649
В Январе-Феврале в сияхской тундре поработай с проживанием в машине. Потом будешь рассказывать.
Аноним 06/08/20 Чтв 12:11:52 593686186
>>593685
Я срал в -40, я знаю что говорю. Ничего в этом такого нет, пиздоболина.
Аноним 06/08/20 Чтв 12:55:18 593693187
>>593686
То есть надо уехать в сухой климат и срать как можно быстрее и мокрее. Так?
Аноним 06/08/20 Чтв 12:56:52 593695188
>>593693
>срать как можно быстрее
Нет, блядь, давить какаху полчаса.
Посрал, вытер, готово, нахуй. Ничего не будет даже в -70.
Аноним 06/08/20 Чтв 13:20:32 593698189
>>593695
За себя не скажу, но многие так делают.
Аноним 06/08/20 Чтв 13:28:26 593699190
>>593675
>Если полетит в сторону сжватого простраства, то и сам сожмется же.
Зачем ему это делать? Блядь, было у корабля скажем 1000км. до цели, стало 250км. Скорость он не меняет. Почему бы при неизменной скорости и сокращённой дистанции не долететь в четыре раза быстрее?
Аноним 06/08/20 Чтв 13:28:41 593700191
>>593686
в -70 посри, засранка.
Аноним 06/08/20 Чтв 14:00:02 593705192
Screenshot20200[...].jpg 36Кб, 676x454
676x454
>>593699
Так расстояние не сократится же. Вот 2д модель с википедии. Если ты не перелетишь этот обрыв через третье измерение, то пройдя вниз по обрыву и поднявшись снова наверх, ты пройдешь те же 1000км. но еще и всрешь энергию на преодоление гравитации, так как гравитация это и есть искривление пространства-времени
Алсо, прочитал что сам корабль в этом пузыре вообще не движется - перемещается сам пузырь. И это еще более непонятно - схуяли ему перемещаться?
Аноним 06/08/20 Чтв 14:18:13 593709193
>>593698
Как? Тужатся по полчаса? Тогда они дауны. Не можешь срать - не мучай жопу. Выходить посрать надо когда хочется срать, а не рассиживаться по часу читая всякую хуйню без толку. Захотел посрать - выдавил личинку, вытер жопу, готово, меньше минуты на все про все.
В космонавты таких глиномесов наверняка не берут, хуй ты рассядешься, от очка улетать будешь без вакуума в очке, а с ним тебе за час все кишки высосать успеет.

>>593700
Надо будет - посру.
Аноним 06/08/20 Чтв 15:23:21 593721194
Аноним 06/08/20 Чтв 15:42:01 593730195
>>593705
Ты продолжаешь не понимать суть пузыря. Никуда перешагивать не надо. Генерируешь его вокруг корабля, он сжимает пространство впереди тебя — предположим, делает так, чтобы один километр впереди тебя стал одним нанометром. Пузырь вместе с кораблём пролетает этот нанометр, по пути сжимая следующий километр, а оставшийся позади нанометр разжимается обратно в километр. Я не понимаю, что тут можно не понимать.
Аноним 06/08/20 Чтв 15:48:18 593732196
>>593721
Смотря как выжить. Глубоководным похуй будет (тем что не сварились)
Это гипотетический класс, и хер знает как он себя ведет. Живи себе в челябе и в ус не дуй, туда никогда никакой торнадовый ураган не заходил, а вот в муриканской долине торнадо станет долина смерти.
Аноним 06/08/20 Чтв 16:06:05 593733197
>>593709
Двачую говноспециалиста.
Аноним 06/08/20 Чтв 19:49:12 593785198
>>593730
>Пузырь вместе с кораблём пролетает этот нанометр, по пути сжимая следующий километр
А куда девается предыдущий сжатый километр? Мудак.
Аноним 06/08/20 Чтв 20:05:14 593791199
>>593785
Прочитай дальше процитированного, шизик, что тебе непонятно?
Аноним 06/08/20 Чтв 20:11:17 593793200
>>593570
>Просветите меня пожалуйста, о том какие химические реакции будет перспективно применять в будущем, кроме метан-кислород.
1. Очевидная замена кислорода на озон, как только научатся стабилизировать.
2. Безуглеродная азот-органика и, мейби, бор-органика. Образуются во всяких экзотических условиях при давлениях в миллионы атмосфер, в наших условиях метастабильны и запасают дохуя энергии.
3. Металлический водород. С ним нихуя не ясно. Может просто чуть плотнее водорода обычного, может мега-йоба с плотностью энергии на кг в десятки раз выше любого топлива.
Аноним 06/08/20 Чтв 20:18:42 593796201
image.png 8Кб, 536x35
536x35
>>593793
1 А есть идеи как стабилизировать? А то ты написал, а это такая же бумажная идея как жечь металлический водород тетракислородом - в теории реально, на деле чет насжимали ну там блестит в алмазном прессе ну хуйзнает кароч
2
>безуглеродная
>органика
Тут я проиграл.
Поясняй давай, лол.
Аноним 06/08/20 Чтв 20:50:14 593799202
Дабы не плодить темы, напишу тут манягипотезу. Скажу сразу, я нуб, поэтому не бейте сильно. Если есть ещё где у кого подобные мысли - сообщите, почитаю. Кароч, как и все школе я учил, что вселенная началась с большого взрыва. Потом узнал, что после взрыва была ещё инфляция, которая потом остановилась. И я тут подумал, а нахуя взрыв перед инфляцией? Представим, что есть бесконечное вечное пространство. И оно заполнено инфляционным полем, то есть постоянно люто бешено расширяется. Настолько быстро, что никакие частицы не могут между собой взаимодействовать. Инфляцию можно назвать антиподом материи и енергии. Это материя с отрицательной массой или вроде того. Где материя и енергия согласно ото пространство-время "вгибает", то инфляция наоборот "выгибает". Но не суть. Инфлиционное поле как и все подвержено квантовым флуктуациям. А так как пространство бесконечно и вечно, то там возможны флуктуации вообще любой величины. И вот в какой-то момент в результате флуктуаций инфляционное поле в определенном объеме ослабло, а частицы "обычные" (с плюсовой массой) появились в большом объеме (напомню, что частицы могут появляться из ничего и пропадать в результате тех же флуктуаций). Далее возможны три варианта. Вариант 1. Массы возникло больше, чем инфляция "вокруг". В таком случае эта область сколлапсирует в сингулярность. Вариант 2. Массы меньше, чем инфляции вокруг. Тогда инфляция будет растягивать область с возникшей массой. Причем с ускорением, поскольку в процессе растягивания области, частицы будут все дальше друг от друга и будут все меньше притягиваться друг к другу. Вариант 3. Масса равна инфляции. Тут либо в результате флуктуаций равновесие все равно нарушится и тогда вариант 1 или 2. Либо флуктуации будут случайно уравновешиваться и такая система будет всегда существовать (а мы ведь говорим про бесконечное пространство, где возможно всё, даже наименее вероятное). И наша вселенная, это вариант 2. Массы возникло немного меньше, чем инфляции вокруг. И инфляция начала вселенную растягивать. Сначала немного, так как частицы вещества были близко друг к другу и сильнее притягивались. А дальше все быстрее. То есть темная энергия это и есть инфляция! И вселенную в конце концов разорвет. То есть наша вселенная не возникла из сингулярной точки. Она сразу возникла в том объеме, который по общепринятой теории большого взрыва она имела после завершения инфляции. Вот только в моей гипотезе инфляция и не заканчивалась никогда.
Аноним 06/08/20 Чтв 20:52:29 593800203
>>593799
Какой еще Енергии, блин. Клавиша "э" явно работает.

Балаган какой-то неудобоваримый.
Почитай про SuSy, может найдешь схожести.
Аноним 06/08/20 Чтв 20:53:02 593801204
>>593709
Я конечно свечку не держал, но мне кажется, что они не тужатся рассчитывая на способность кишечника самопроталкивания.
Аноним 06/08/20 Чтв 20:56:42 593802205
>>593801
Зависит от стула.
Краем глаза слышал, что у космонавтов на орбите он быстро становится жидким на время полета.
Кажется, что брехня, но может это специально и так удобней сортиру?
Аноним 06/08/20 Чтв 21:08:14 593804206
>>593800
Ну коснозычен я, что поделать. Основная идея, что инфляция была до большого взрыва, темная энергия - это и есть инфляция, вселенная не была сжата в точку вначале.
Аноним 06/08/20 Чтв 21:08:55 593806207
>>593800
Susy уже гуглю, спасибо.
Аноним 06/08/20 Чтв 21:24:39 593812208
>>593799
Насколько я слышал, то "взрыв" это и есть инфляция, продолжение которой мы наблюдаем в виде ТЭ. Что было до гадать бессмысленно тысячи вариантов. Но одно понятно что что-то было. И это БВ не из точки, а из малого объёма пространства. Так же есть гипотеза, что ценой за инфляцию пространства стало возникновение материи. Но это гипотеза.

Алсо. Манягипотезы это конечно хорошо, но даже если они верны, не факт что твои маняследствия валидны.

Насчёт флуктуаций. Статистически вероятнее возникновение лишь земли и фотонов которые сформируют картину сложной вселенной, чем флуктуация всей вселенной.
Аноним 06/08/20 Чтв 21:57:41 593814209
>>593812
Я кое-что нагуглил, но в силу скудности физических знаний все эти "ложные вакуумы" не сильно понятны. Насколько я понимаю, эти модели предполагают огромный бесконечный "океан" постоянно "инфлуирующего" пространства (мультивселенная), в котором иногда образовываются пузырки-вселенные, которые, однако, окружающая, кхм... "вода" рано или поздно разрывает.
Аноним 06/08/20 Чтв 22:37:19 593819210
IMG202008061706[...].jpg 119Кб, 788x960
788x960
Двач ответь. Куда девается материя со звёзд? Правда ли что если у меня есть 10 звезд, через время они умирают но потом под действием силы притяжения они снова запускают старые процессы, стают теми же 10-ю звёздами и всё по новой? Если нет то что тогда происходит?
Аноним 06/08/20 Чтв 23:39:57 593839211
>>593819
Ебать, картинка прям в шапке треда. Ты совсем поехавший?
Аноним 07/08/20 Птн 00:27:24 593857212
>>593839
Но там же ничего про количество не написано долбоёб
Аноним 07/08/20 Птн 00:39:11 593860213
image.png 59Кб, 1000x657
1000x657
>>593819
>под действием силы притяжения они снова запускают старые процессы
Выгоревший водород не восстанавливается, в итоге все процессы заглохнут около железа-56.
Аноним 07/08/20 Птн 11:15:30 593928214
Гиперциклон1.png 1108Кб, 1920x1028
1920x1028
>>593721
Ближе к полюсам если жить то думаю будет нормально. При условии что планета осиливает на экваторе нагревать поверхностный слой океанов до +50, то какие нибудь умеренные климатические зоны вполне себе субтропиками будут, не исключено что тамошней зимой в условной местной России разве что в самых северных частях опускало бы до -10, -25 и то не факт и желательно подальше от побережья не считая побережья тамошнего океана около полюсов. А вот около экватора лютый писец происходить будет, даже без гиперциклонов. Причем такой что на кораблях будет весьма сложно взаимодействовать с противоположным полюсом. Да и авиационным будет крайне сложно, совершенно точно придется облетать долбаные гиперциклоны, но там и обычных будет дофига и больше, да и в целом атмосфера куда более бурной будет. Однако если на планете будет меньше океанов, чем у нас на Земле, то ещё более менее ничего может быть, не считая того что большая часть континентов будет хреновым горячим сухарем без осадков.
Аноним 07/08/20 Птн 11:18:32 593929215
image.png 324Кб, 800x450
800x450
>>593928
Зато облегчится выход в космос. (Это была шутка юмора есличо)
Аноним 07/08/20 Птн 11:21:43 593930216
>>593929
Теоретически можно приспособить йоба ветра гиперциклонов для генерации кучи энергии на йоба ветриках и притом все твердо будет, если конечно в принципе можно сделать инфраструктуру что стабильно выдержит ветра за сотню, полторы метров в секунду. А так же возможно это можно использовать для офигенно долгого планирования какой то авиационной техники. Но ещё вопрос а нужно ли.
Аноним 07/08/20 Птн 12:13:09 593943217
004.png 10Кб, 351x311
351x311
>>593796
>А есть идеи как стабилизировать?
Теоретически - сверхвысокой очисткой.
Почему вообще озон рвётся? Потому что рядом проплывала залётная молекула какого-нибудь оксида, с которой ему вроде бы уже и не надо реагировать, но молекула поляризована и этими остаточными расшатала "вилочку", а вилочка порвалась. При этом образуется так дохуя энергии, что соседние молекулы озона тоже рвутся - и здравствуй цепная реакция.

Может быть можно стабилизировать внешним электростатическим или электромагнитным полем. Чтобы диполные моменты не шароёбились, а изначально стояли бы по стойке смирно. Такой катализатор-наоборот.

>Поясняй давай, лол.
Пикрелейтед.
Трёхвалентные атомы тоже могут в цепочки и колечки. При этом энергия там запасается не за счёт избытка водорода, а за счёт как бы деформированности этих цепочек. Они образуются при давлениях, когда энергия давления сопоставима с энергией химической связи, но после сброса давления сразу не рвутся, а как бы растягиваются.
Топовое вещество пока что - кубический гош-азот. Запасает до 33МДж/кг, что является наивысшим из известных неядерных материалов, превосходя даже пару литий-фтор и бериллий-кислород.

Есть ещё пока сугубо гипотетическая тема про двухвалентные атомы, способные образовывать либо колечки (тот же тетракислород), либо длинные замкнутые спиральки с плохо понятными квантовыми эффектами. Хуй его знает, может быть при терапаскалях можно свернуть кислород в такую хрень.

Всё это пока - крайне малоизученные области химии. Мы пока считаем электростатические потенциалы отдельных атомов и более-менее раскладываем их на пространственных структурах углеродной органики, но не рассматриваем как механическую и квантовомеханическую систему с упругими деформациями. Как завезут суперкомпьютеры на тензорных процессорах - так начнётся крупномасштабное брутфорс-моделирование всей этой хуйни, и более-менее начнётся химия v2.0 по этой теме. Может быть даже до внутренних электронов тяжёлых ядер докопаются - то, на что лично я дрочу. Энергия связи там яебу, но в нормальных химических связях они не участвуют. А вот если свернуть молекулу раком системой рычагов - может и получится это дело расшатать.
Аноним 07/08/20 Птн 12:24:02 593946218
image.png 203Кб, 929x1466
929x1466
>>593943
>33МДж/кг, что является наивысшим из известных неядерных материалов, превосходя даже пару литий-фтор и бериллий-кислород.
Как пикрил считать?
Аноним 07/08/20 Птн 12:26:23 593950219
image.png 12Кб, 351x311
351x311
>>593943
Что это за омск? Какая тут у углерода в кружочке валентность?
Аноним 07/08/20 Птн 12:31:24 593953220
image.png 13Кб, 351x311
351x311
>>593950
А, я кажись понял, каждый N3 это сам по себе цикл.
Аноним 07/08/20 Птн 13:39:04 593972221
>>593785
Сначала сжимается километр. Потом сжимается километр одновременно с разжатием нанометра. Потом сжимается километр одновременно с разжатием нанометра. Потом сжимается километр одновременно с разжатием нанометра. Потом сжимается километр одновременно с разжатием нанометра. Потом сжимается километр одновременно с разжатием нанометра. Потом сжимается километр одновременно с разжатием нанометра. Потом сжимается километр одновременно с разжатием нанометра. Потом сжимается километр одновременно с разжатием нанометра. Потом сжимается километр одновременно с разжатием нанометра.
В конце разжимается нанометр.
Так понятно?
Аноним 07/08/20 Птн 13:43:27 593974222
>>593972
Мне кажется это бесполезно. Некоторые бывают полностью непробиваемыми как плоскоземельщики и антимаскеры.
Аноним 07/08/20 Птн 14:09:37 593981223
140142532-kyiv-[...].jpg 187Кб, 1300x778
1300x778
На Луне как известно трудно передвигаться и приходится прыгать нелепыми прыжочками. А что если использовать гироскутеры? Если колесо пошире сделать то должен работать и на пересечённой местности.
Аноним 07/08/20 Птн 14:13:42 593983224
>>593981
Абсолютно точно нет.
Если на земле при отказе этой хуйни по любой причине ты отделаешься синяком или царапиной, то на луне порванный скафандр или разбитое забрало это 100% смерть.
Никто не будет так огурцами рисковать.
Аноним 07/08/20 Птн 14:16:41 593984225
004.png 17Кб, 800x798
800x798
>>593946
По-хорошему надо каждую отдельную атомную связь в электронвольтах считать, потом через авогадро в дж/моль переводить и оттуда в килограммы. Причём энергия выделяется не только при окислении водорода или углерода с кислородом, как в школе учили, но и при разрыве связей сложной молекулы. А в некоторых она наоборот поглощается. И вообще реакция идёт ступенчато, и промежуточных веществ в ней может быть овердохуя.

>>593953
Всё правильно. Там вообще своя атмосфера в экзотической химии этой.
Аноним 07/08/20 Птн 14:22:10 593986226
>>593983
А скафандр и не нужен, нужна жёсткая капсула в которой сидит огурец и манипуляторы снаружи. И гироколесо снизу.
Аноним 07/08/20 Птн 14:27:06 593989227
image.png 1389Кб, 1200x794
1200x794
>>593986
Не выдумывай решения для несуществующих проблем.
Аноним 07/08/20 Птн 14:28:42 593992228
>>593981
Гироскутер работает на компенсации притяжения реактивным моментом. Ты наклоняешься вперёд, а колесо едет и пытается разнаклонить тебя назад. Пока это в равновесии - ты модно катаешься, и управляющий угол требуется очень маленький.

На Луне гравитация слабенькая, поэтому все подобные фишки будут работать через жопу. Тебе надо будет либо охуенно нагибаться, либо колесо будет тебя переворачивать. Прибавь к этому возможность подскочить на любой кочке, а в полёте с гироскутером совсем плохо всё.

Вообще поиграй в огурцов дабы прочуствовать все проблемы скоростных лунных роверов. Там сложно не разогнаться - сложно не улететь и не начать кувыркаться как ебанутый. И лучше всего себя чувствуют максимально распластанные роверы с большими тонкими колёсами.
Аноним 07/08/20 Птн 15:18:06 594010229
>>593989
Не холодно, в открытом-то?
Аноним 07/08/20 Птн 15:21:01 594012230
>>594010
На солнце же, наоборот напекает, потому пляжный зонтик и раскрыли.
Аноним 07/08/20 Птн 15:30:27 594014231
>>593814
Послушай Верходанцева.
Аноним 07/08/20 Птн 15:37:36 594016232
>>593814
Не слушай этого >>594014 анона и всяких верхнедаунцевых.
Аноним 07/08/20 Птн 15:40:49 594019233
>>594012
Похоже, его ветром перевернуло.
Аноним 07/08/20 Птн 15:45:22 594022234
>>594019
Да, в ветренную погоду неудобно. То флаг тебе колышется, то вообще зонтик выворачивает. Ладно хоть под дождь не попали когда гостили.
Аноним 07/08/20 Птн 15:52:43 594025235
Аноним 07/08/20 Птн 16:52:24 594050236
image.png 106Кб, 250x399
250x399
>>593981
У него там зеркало заднего вида на очках? Не боится его потом из башни выколупывать после падения?
Аноним 07/08/20 Птн 16:52:26 594051237
>>593981
К анонам выше добавлю, что при пониженной гравитации проблемой является также трение. Давление на опору ниже, и твоя хуйня скользит по реголиту как по льду. В луноходах с этим туго. Единственным решением является увеличение массы и площади касания.
Аноним 07/08/20 Птн 16:54:54 594053238
image.png 326Кб, 602x371
602x371
>>594051
>Единственным решением является увеличение массы и площади касания.
'sup
Аноним 07/08/20 Птн 17:01:32 594057239
>>594053
Двачую инженера.
Аноним 07/08/20 Птн 17:02:28 594058240
>>594053
>>594057
Можно на системе посадки сэкономить. Литобрейкать будет.
Аноним 08/08/20 Суб 14:32:53 594278241
Аноним 08/08/20 Суб 14:34:54 594279242
>>594278
>Верходанов
Если он такой умный, то почему он такой мертвый в 55 лет?
Аноним 08/08/20 Суб 16:29:57 594292243
Самый заезженый вопрос.
Если в одну сторону направить ракету со скоростью 0.9 скорости света и в противоположную сторону направить такую же ракету, то с какой скоростью они будут двигаться относительно друг друга?
Мне кажется, что их относительная скорость будет 1.8с, но если я буду сидеть на первой ракете, то как только стартанет вторая ракета, для меня она тут же исчезнет - ведь свет от нее не будет успевать долететь ко мне. А раз относительно меня вторая ракета "не существует", то и на относительную скорость поебать. Я прав?
Аноним 08/08/20 Суб 16:32:18 594293244
>>594292
>то как только стартанет вторая ракета, для меня она тут же исчезнет - ведь свет от нее не будет успевать долететь ко мне.
М, нет. Он будет сильно покрасневшим, но тебя догонит
Аноним 08/08/20 Суб 16:33:43 594294245
>>594293
почему? Скорость света же 1с, а я относительно корабля лечу со скоростью 1.8с
Аноним 08/08/20 Суб 16:36:06 594295246
>>594292
>Если в одну сторону направить ракету со скоростью 0.9 скорости света и в противоположную сторону направить такую же ракету, то с какой скоростью они будут двигаться относительно друг друга?
0.9944с
>Мне кажется, что их относительная скорость будет 1.8с, но если я буду сидеть на первой ракете, то как только стартанет вторая ракета, для меня она тут же исчезнет - ведь свет от нее не будет успевать долететь ко мне. А раз относительно меня вторая ракета "не существует", то и на относительную скорость поебать. Я прав?
Нет.
Глянь вот этого Висоса: https://youtu.be/ACUuFg9Y9dY
Популярно поясняет, наглядно, может поможет.
Аноним 08/08/20 Суб 16:38:04 594297247
Аноним 08/08/20 Суб 19:43:44 594323248
>>594294
А скорость света относительно тебя всё равно будет 1 с
Аноним 08/08/20 Суб 19:54:02 594325249
>>594323
Мне кажется, ему видео Майкла Стивенса помогло. Понимание релятивистской механики и почему всегда с это с безотносительно скорости приходит не сразу.
Аноним 08/08/20 Суб 20:51:51 594331250
>>594295
>>594297
Подорванный какой-то. Смотреть невозможно.

мимокрок
Аноним 08/08/20 Суб 21:02:34 594335251
image.png 999Кб, 1700x1390
1700x1390
>>594331
В самый раз для современных зумеров.
Ну а кого еще посоветовать-то?
Наше Всё вроде не делал про это.
Веритася, Смартер, Моулд, Скотт по этой теме не занимаются.
PBS сложновато будет да и не переводили его.
Хотя ладно, вдруг язык не помеха: https://youtu.be/msVuCEs8Ydo
Картинки с поняшками помогут.
Аноним 09/08/20 Вск 02:04:51 594391252
У женщин менструальный цикл на луне или Марсе с ума не сойдёт?
Аноним 09/08/20 Вск 05:18:14 594411253
>>594391
Не сойдет. Он от луны не зависит.
Аноним 09/08/20 Вск 15:18:37 594455254
Как обнаружили и обнаруживать широкие атмосферные ливни?
Аноним 09/08/20 Вск 18:03:08 594500255
Безымsasdasdsad[...].png 25Кб, 1665x826
1665x826
Посоны, а реально ли поднимать грузы на орбиту узконаправленным магнитным лучом? Сделать типо магнитный космический лифт. На орбите будет ядерный реактор как у подлодки. Он даст электричество магниту. Эдакий трактор-бим.
Аноним 09/08/20 Вск 19:28:02 594515256
>>594500
>узконаправленным магнитным лучом
Такого не бывает.
Аноним 09/08/20 Вск 20:24:01 594520257
Можно ли за счёт лазеров сжигать или разогревать воздух на пути ракеты, облегчая взлёт?
Аноним 09/08/20 Вск 20:30:11 594521258
>>594520
Не поможет, если ты разогреешь воздух, он увеличится в объеме, в итоге создав волну расширения и дуя на ракету, как если бы ты перед ней взрывал взрывчатку и дул на нее взрывной волной, мешать только будет. Это не говоря о том, что особо нормально не сфокусируешь на воздухе.
А вот на жопку ракеты можно лазером светить и она от этого сможет летать. https://youtu.be/XhUasBcoj-Q
Аноним 09/08/20 Вск 20:30:49 594522259
>>594455
Слово широкие намекает на то, что тебе нужны большие площади для детектирования оных. Так же нужны детекторы частиц на этой площади. Так же сложная математика, осуществимая в реальные сроки на компьютере. Всё это должно быть собрано в систему на подобие черенковских телескопов, обеспечивать нное количество рабочих мест и работать до тех пор, пока институт даёт деньги.
Аноним 09/08/20 Вск 20:31:58 594523260
>>594522
Ты не ответил вообще никак на заданный вопрос.
Аноним 09/08/20 Вск 20:47:16 594527261
>>594523
А в чём вопрос? Как обнаружили это к истории науки, а как обнаруживают я тебе сказал черенковскими телескопами и орбитальными гамм-телескопами.
Аноним 09/08/20 Вск 20:48:14 594528262
>>594527
Так их получается почти не видят?
Аноним 09/08/20 Вск 20:52:46 594530263
>>594528
Кого их. Ливни? Или частицы вызывающие ливень? Если их обнаружили, значит их видят. И с каждым новым оборудованием всё лучше и лучше.
Аноним 09/08/20 Вск 20:53:48 594531264
>>594530
Ливни.
Так что за детекторы, где они?
Аноним 09/08/20 Вск 21:35:55 594545265
почему плазма считается отдельным аггрегатным состоянием?
это же просто обычный заряженный газ
Аноним 09/08/20 Вск 22:00:17 594548266
>>594545
Ведёт себя не как газ.
Аноним 09/08/20 Вск 22:26:48 594550267
>>594455
Была (или есть?) краудсорсинговая программа по поиску этих ливней. Волонтеры ставят апп на старый смарт, заклеивают ему камеру, и значит любой проблеск это частица. Если на куче волонтерских смартов одновременно есть детект - значит обнаружен ливень, по распределению можно и параметры вызвавшей его частицы посчитать.
Аноним 09/08/20 Вск 23:13:49 594552268
Можно ли покрыть спутник водой, чтобы его спектроскопический профиль изменился? Типа выпускаем из капсулы воду так чтобы она обволокла спутник. Или же спектр покажет одно, а радар через воду пробъется и всё равно покажет металл?
Аноним 10/08/20 Пнд 01:24:05 594558269
>>594552
Вода моментально испарится, о каком покрытии ты вообще говоришь?
Аноним 10/08/20 Пнд 02:03:16 594564270
>>594552
Водой - нет, она испарится. Неким устойчивым к вакууму метаматериалом - до определенной степени да, с кучей оговорок. А нахуя тебе это?

Но надо понимать, что у спутника и еще требования есть, в ЭВТИ их не зря заворачивают. Ещё приборы не покроешь.
Аноним 10/08/20 Пнд 02:14:28 594566271
>>594564
Я так понял нужно покрыть чем-то таким невызывающим подозрения. На орбите Земли по температуре плавления какое-нибудь олово подойдёт, но спутник в капле олова не выживет, да и не выглядит оловянная капля натурально.

Вода могла бы сработать вдали от Солнца
Аноним 10/08/20 Пнд 02:57:58 594569272
>>594564
>Неким устойчивым к вакууму метаматериалом
Любая жидкость в вакууме быстро испарится, или нет? Я так понимаю, в жидкости межмолекулярные связи слабые, и молекулы будут быстро "выпрыгивать" и улетать нахуй, так?
Аноним 10/08/20 Пнд 03:23:05 594571273
>>591947
Тащемто там недавно прохавали что мозг получает гораздо больше инфы в итоге с глаза чем собственно глаз собирает. Тоесть в крации если: глаз видит как камера нокии 200х300 а в мозгу все это апскейлится до 4к. Так что по твоему вопросу - похуй на глаза, главное как это будет обрабатывать мозг.
Аноним 10/08/20 Пнд 03:30:12 594572274
>>594571
Не апскейлится, а клеится в панораму в реальном времени, местами со спецэффектами постобработки. Тема интересная, но с уменьшением размеров слабо связанная.
Аноним 10/08/20 Пнд 07:17:50 594580275
Аноним 10/08/20 Пнд 08:37:14 594589276
Если в лунные кратеры щедро налить незамерзайки, на луне будут моря из незамерзайки? Или она испарится как вода?
Аноним 10/08/20 Пнд 10:44:27 594598277
>>594589
Изопропанол-то? Конечно испарится.
Аноним 10/08/20 Пнд 12:49:47 594610278
>>594569
Любая испаряется, но некоторые очень медленно, вплоть до пренебрежимых в некоторых ситуациях значений. Сложноорганические вакуумные смазки, например, или еще ионные жидкости (расплавленные соли)
Аноним 10/08/20 Пнд 12:55:25 594612279
>>594566
Замаскировать под естественный объект что ли? Естественных объектов на орбите Земли нет, кроме Луны, да и любой спутник и кусочек мусора ведут от запуска до свода.
Аноним 10/08/20 Пнд 13:19:10 594616280
>>594612
А если замаскировать спутник под Луну?
Аноним 10/08/20 Пнд 13:35:22 594617281
>>594616
На Луну даже твоя мамаша не потянет.
Аноним 10/08/20 Пнд 18:45:59 594701282
Излучение Хокинга подразумевает, что черная дыра однажды испарится. Точнее испарится вещество вокруг неё, то, что она выбрасывает. А что будет с веществом, которое внутри неё находится, которое за горизонтом событий? Оно куда денется? Оно же не испарится? И оно же никогда не сможет исчезнуть, ведь там протонов нет и распадаться нечему?
Аноним 10/08/20 Пнд 22:39:05 594731283
>>594701
Большой разрыв раньше настанет.
Аноним 11/08/20 Втр 11:32:07 594791284
>>594531
Фотодетекторы. Регистрируют кванты света от распада всяких нестабильных частиц. Компьютер всё это анализирует и воссоздаёт какая частица от какой распалась. В итоге получаем параметры исходной частицы.
Аноним 11/08/20 Втр 15:21:22 594847285
>>594791
Ясно, моченые погадали. Лишь бы гранты продолжали выделять.
Аноним 11/08/20 Втр 15:22:00 594848286
триумф зеленого[...].jpg 114Кб, 800x600
800x600
Аноним 11/08/20 Втр 15:59:49 594861287
15918759589960.png 680Кб, 600x804
600x804
В чем разница между самым обычным вакуумом, т.е. пространством, в котором насколько я понял нет никаких частиц, ну т.е. вообще нихуя в нем нет, пусть такого идеального вакуума может и не бывает, но все же и тем, что находится за пределами пространства?
Вот ученые в говне моченые говорят, что за пределами вселенной пространство заканчивается и при этом оно еще и расширяться умудряется. И это нихуя за пределами пространства это не вакуум. А тогда что это, блять? Инбифо ничто.
Аноним 11/08/20 Втр 16:01:16 594862288
>>594861
Ну да, ничего.
Возьми дискету на 1.44Мб. Что находится по адресу второго мегабайта?
Аноним 11/08/20 Втр 16:05:24 594863289
>>594862
Так в чем разница?
Аноним 11/08/20 Втр 16:09:20 594864290
>>594863
В вакууме что-то может быть. А "нигде" ничего не может быть. Этого нет. "Там" не существует.
Аноним 11/08/20 Втр 16:11:55 594865291
>>594864
>В вакууме что-то может быть.
А может и не быть, так? Так в чем тогда разница, я не понимаю, и то и другое ничего.
Аноним 11/08/20 Втр 16:15:55 594866292
>>594865
Вакуум-то не ничего.
То что ты называешь вакуумом - это пространство-время Вселенной, можешь считать ее тканью мироздания, эфиром, пердежом Иисуса или мегабайтами.
То что "может не быть" не существенно.
Важно что "может быть".
Этого "там" нет, там ничего не может быть и все, это не пустота.
Ты думаешь, это 0, как противоположность 0/1 "вакуума", а это <null>, void, nullptr.
0/1 - данные. 0 тоже данные.
Там нет данных, если про информатику говорить
Аноним 11/08/20 Втр 16:16:17 594867293
>>594861
>за пределами пространства
У пространства нет пределов.
Аноним 11/08/20 Втр 17:03:52 594875294
Аноним 11/08/20 Втр 17:05:56 594877295
154799718116621[...].jpg 45Кб, 495x268
495x268
>>594875
>мой мясной мозг чего-то не понимает, значит этого не может быть
Аноним 11/08/20 Втр 17:59:34 594882296
Где почитать самую примитивную инфу про гравитацию планет и между планетами? Ну например если сильно разогнать спутник, то в каком случае он будет летать по сильно вытянутой орбите, а в каком улетит от Земли навсегда? И еще, реально ли спутник "остановить" где-нибудь в космосе между планетами или он будет всё время к кому-то притягиваться?
Аноним 11/08/20 Втр 18:10:28 594886297
>>594882
Левантовский В. И. - Механика космического полёта в элементарном изложении.
Аноним 11/08/20 Втр 18:22:44 594891298
7640215-1.jpg 184Кб, 900x1200
900x1200
Аноним 11/08/20 Втр 18:27:18 594893299
37363936.jpg 100Кб, 321x500
321x500
Аноним 11/08/20 Втр 18:57:29 594902300
Во что превратится говно, если оставить его на поверхности Луны на пару лет?
Аноним 11/08/20 Втр 19:06:01 594903301
>>594902
В прах с сохранением общей формы.
Вся вода быстро сублимируется, останется очень легкий и хрупкий остов из мертвых микроорганизмов и остатков пищи светло-серого цвета и зольной консистенции, как хорошо прогоревшее бревнышко.
Аноним 11/08/20 Втр 19:14:56 594904302
Аноним 11/08/20 Втр 19:21:28 594906303
>>594904
Собрался посрать на луне?
Аноним 11/08/20 Втр 19:35:07 594910304
>>594886
Охуенно, благодарю.
Аноним 11/08/20 Втр 23:29:52 594932305
Поясните за монополь, я никак не могу представить себе как это отдельно полюс магнита
Аноним 12/08/20 Срд 00:26:11 594937306
>>594932
Не думай об этом как о полюсе магнита, а просто как об отдельном заряде, очень похожим на электрический. Ты же не думаешь об электрическом заряде как об отдельном конце проводника с током?

Уравнения Максвелла абсолютно симметричны для электрических и магнитных зарядов, если бы монополи существовали, они бы вели себя точно так же, как сейчас себя ведут электроны и протоны. Вся разница в том, что в определениях все слова «электрический» поменялись бы на «магнитный», и наоборот.

Вместо электрических проводов были бы магнитные проводники, по которым тёк бы магнитный ток, и создавал бы вокруг себя электрическое поле. Свил магнитный проводник вокруг сердечника — получил магнитоэлектрик (как электромагнит, только наоборот). Всё один в один.

Просто в нашу вселенную магнитных монополей походу не завезли и магнитное поле не существует само по себе, а только порождается электрическим током, и поэтому кажется, что оно как бы вторично по отношению к электрическому, но на самом деле они полностью равноправны.
Аноним 12/08/20 Срд 01:15:46 594944307
>>594866
Программист, ты же гуманитарий, ты знаешь? Нахуй ты свои железки проецируешь на реальный мир.
Аноним 12/08/20 Срд 08:02:36 594960308
>>594937
Стало на йоту проще, но недостаточно для принятия, что еще можно почитать?
Аноним 12/08/20 Срд 10:33:07 594967309
>>594861

Размер вселенной конечен. Да она расширяется, но в какой-то момент времени она имеет размер. И вселенная замкнута. Ты не можешь покинуть землю если будешь просто бежать вперед или плыть. Ты никогда не увидишь "край мира" или "великую стену за которой ничего нет". Хотя земля конечна и имеет определенный размер и объем. Ты просто будешь бегать по глобусу кругами и возвращаться в то же место откуда пришел. Расширяется при этом земля или нет - тебе абсолютно похуй. Ты находишься на замкнутой поверхности. теоретически ты можешь улететь на ракете, но для этого тебе нужно двигаться по третьей оси.

Чтоб покинуть вселенную нужно по какой-то четвертой оси двигаться, но мы так не умеем, поэтому даже если бы вселенная была размером с солнечную систему мы бы все равно ее не смогли покинуть. Вояджер бы вылетел с одной стороны и прилетел к нам с противоположной, поскольку пространство замкнуто.

Так же как на земле ты не можешь начертить линию и сказать что вот это край (Потому что где бы ты не находился на сфере - во все стороны есть еще тысячи километов поверхности. Любая точка на сфере может считаться "верхушкой"), так же и во вселенной нельзя долететь до края или сказать, что вот там уже ничего не будет.

Любая точка вселенной может считаться центром вселенной потому что из этой точки во все стороны одинаково дохуя пространства

Ок, ты сейчас спросишь "И все же земля имеет объем и ее можно покинуть на ракете и посмотреть со стороны. Что если покинуть вселенную?" В нашем понимании, ты не сможешь ее покинуть. Чтоб находиться за ее пределами, нужно чтоб там было пространство, а там его нет. Ты держишь в руках книгу на 100 страниц, а тебя просят прочитать что находится на 200 странице. Ее просто нет. Там не чистый лист - страницы нет вообще. У тебя в телефоне записаны номера только бати и мамки, а тебя просят сказать какой там телефон у Гоши Говнякина в твоем телефоне. а такой записи просто не существует.
Ничего за пределами пространства нет и быть не может потому что иначе это бы было пустое пространство.
Единственным способом выйти за пределы вселенной - это попасть в другую вселенную.
Но "между ними" ничего нет.
Представь что ты расчертил на земле сетку как для морского боя и каждый квадрат в этой сетке 1х1 метр. Ты можешь перемещаться по квадрату, ты можешь пойти в другой квадрат, но ты не можешь находиться так, чтоб не стоять ни на каком квадрате, если ты находишься внутри сетки. У сетки нет понятия "между клетками". Так что ты или в этой вселенной или в другой. Но не во вселенной ты быть не можешь, так как буквы могут быть на какой-то странице, но если страницы нет, то буквы не могут висеть в воздухе. Если буквы написаны, то они написаны на чем-то, а "ни на чем" они не могут быть написаны.
Аноним 12/08/20 Срд 10:37:23 594968310
>>594967
Интересно, мы это понимаем, или просто приняли как данность, а мясной мозг такое осознать не может?
Аноним 12/08/20 Срд 11:00:39 594972311
Возможно ли существование геев-ниггеров из далекого космоса?
Аноним 12/08/20 Срд 11:04:04 594973312
Аноним 12/08/20 Срд 11:09:31 594974313
Аноним 12/08/20 Срд 11:24:12 594975314
Аноним 12/08/20 Срд 16:49:53 595003315
>>594967
Но откуда-то начался большой взрыв? Как эту точку начала задетектить? Она и будет центром!
Аноним 12/08/20 Срд 16:50:54 595005316
>>595003
Анон... от везде начался. Везде центр, если говорить в пространственных координатах.
Аноним 12/08/20 Срд 16:51:02 595006317
>>594967
А вот хуй! Ты сравниваешь Землю со вселенной, но вселенная плоская.
Аноним 12/08/20 Срд 16:52:32 595008318
>>595006
>вселенная плоская
Это не так.
Все вокруг трехмерное.
Шах и мат.
Нечего чушь пороть, которая так легко опровергается.
Про плоскую землю бы сказал - другое дело.
Аноним 12/08/20 Срд 18:49:53 595022319
>>595008
Я недавно слушал Попова и Сурдина и они говорили, что вселенная плоская, но никак не шарообразная.
Аноним 12/08/20 Срд 19:02:38 595023320
image.png 517Кб, 800x439
800x439
>>595022
А если Марка Саржента послушать, то он будет говорить, что земля плоская, а не шарообразная.
Аноним 12/08/20 Срд 19:18:34 595024321
>>595022
плоская это в смысле не форма, а отсутствие искажений, равномерность
Аноним 12/08/20 Срд 19:19:29 595025322
>>595024
А по форме шарообразная.
Аноним 12/08/20 Срд 20:01:41 595035323
>>594937
Что то я не понимаю. Разве может быть магнитный проводник или магнитный ток? Разве и тот и другой не является электромагнитный проводник и электромагнитный ток? Разве ещё не в 19 веке выяснили, что магнитное поле тождественно электрическому? Разве это не одна и та же сущность?
Аноним 12/08/20 Срд 20:10:23 595037324
>>595025
Вы что на гипершаре живёте? Одумайтесь.
Аноним 12/08/20 Срд 20:25:39 595039325
>>593928
Придётся делать сверхвысотные самолёты под высоту 40-50 км, чтобы летать над ураганами.

Или исхитряться облетать ураганы на сверхзвуковых самолётах обычных.

Или сделать стратосферные дирижабли.

Или возить грузы на другой полюс на подлодках.

Какой вариант ни возьми — звучит охуенно
Аноним 12/08/20 Срд 20:29:18 595042326
>>595039
>Или возить грузы на другой полюс на подлодках
А хуй тебе.
Океаны по 50 градусов, приедет подлодка к назначению, вскроют, а там томленое мясо по-мичмански, матросская тушеночка
Аноним 12/08/20 Срд 20:32:00 595045327
>>595042
Да это они сверху 50 градусов, пониже там и 20 не будет.
Аноним 12/08/20 Срд 20:33:38 595046328
>>595035
Есть электромагнитные явления, есть чисто электрические и чисто магнитные. Никто же не говорит про электромагнитный заряд протона/электрона.
Аноним 12/08/20 Срд 20:36:09 595047329
>>595046
Ликтрический заряд, полюс северный
Протон в ядре лежит, непомеренный
Глюоны, кварки - тяжкий груз.
Аноним 12/08/20 Срд 20:39:28 595048330
>>594515
Называется мазер
Аноним 12/08/20 Срд 20:47:13 595049331
>>595048
Это микроволны, а не магнетизм.
Аноним 12/08/20 Срд 20:50:30 595052332
>>595039
Годные сюжеты для книжонки. Я бы почитал.
Аноним 12/08/20 Срд 21:08:01 595056333
>>595046
Кто не говорит? Электромагнитное взаимодействие. Его электромагнитный квант - электрон. Переносчик электромагнитного взаимодействия - фотон.
Аноним 12/08/20 Срд 22:53:51 595078334
>>595037
Лично я живу в гиперболическом пространстве
Аноним 12/08/20 Срд 22:55:39 595079335
>>595078
А я в алкоголическом.
Но эти наши с тобой шизоидные восприятия слабо влияют на реальность.
Аноним 12/08/20 Срд 23:03:51 595081336
>>595056
Электромагнитный квант это бред собачий, электромагнитного заряда не бывает.

Электрон имеет только электрический заряд и не имеет магнитного, равно как и мюон, тау-лептон, кварки, а также их античастицы. Но в теории частицам с ненулевым магнитным зарядом ничего не мешает существовать.

>>595035
>магнитное поле тождественно электрическому? Разве это не одна и та же сущность?
Нихуя подобного. Электромагнитное поле это комбинация электрического и магнитного полей, которые изменяются по одним и тем же законам и порождают друг друга при определенных условиях, но само электрическое и магнитное поля это полностью отдельные его компоненты.

В электрическом проводнике движутся электрические заряды, и вокруг создается магнитное поле, а в магнитном двигались бы магнитные, и вокруг бы создавалось электрическое поле.

И наоборот — при внесении магнитного проводника в изменяющееся электрическое поле магнитные заряды бы приходили в движение и в проводнике создавался бы магнитный ток. Всё абсолютно симметрично нашему миру с электрическими зарядами.
Аноним 13/08/20 Чтв 00:02:57 595096337
>>595081
>Электрон имеет только электрический заряд и не имеет магнитного
А что тогда такое кулоновский барьер, как не проявление магнитных свойств электрона?

Ни про какие электрические или магнитные поля не слышал, знаю только об одном из 4х взаимодействий. И теория и эксперимент говорит о электромагнитном поле, заряде и корпускуле-волне.
И ты так говоришь "магнитный заряд, то же что и мюон", как будто поколений электрона доподлинно известно не 3, а больше.
Аноним 13/08/20 Чтв 00:45:51 595107338
>>595096
>А что тогда такое кулоновский барьер, как не проявление магнитных свойств электрона?
Кулоновский барьер это чисто электрический феномен, магнитные поля тут ни при чем. Хуй знает, где ты там нашёл что бы то ни было магнитное.

>Ни про какие электрические или магнитные поля не слышал, знаю только об одном из 4х взаимодействий.
Которое в свою очередь является проявлением единого электрослабого взаимодействия, а возможно, и сильное туда же входит, но почему-то ты его отдельно рассматриваешь. С чем связано такое двоемыслие?

>И ты так говоришь "магнитный заряд, то же что и мюон", как будто поколений электрона доподлинно известно не 3, а больше.
Вообще не понял, что написано в этом предложении, шизофазия какая-то. Какой мюон, какие поколения электронов, о чём ты?

Лептонов известно три поколения, и у всех них электрический заряд равен единице, а магнитный заряд равен нулю. Кварков известно три поколения, и у всех них электрический заряд кратен 1/3 е, а магнитный заряд тоже равен нулю. Это разные квантовые числа, электрический и магнитный заряд независимы друг от друга, просто у всех наблюдаемых частиц магнитный заряд нулевой.

Тем не менее, ничего не мешает существовать частицам, у которых имелся бы либо только магнитный, либо как электрический, так и магнитный заряды. То, что в реальности их не наблюдается, еще не означает, что их не может существовать гипотетически.
Аноним 13/08/20 Чтв 01:10:50 595112339
orion-62.jpg 47Кб, 400x292
400x292
unnamed.jpg 82Кб, 512x336
512x336
почему за 15+ лет полетов нет фотографии в широком доступе этой необъятной йобы https://space.skyrocket.de/doc_sdat/orion-5_nro.htm

>These satellites feature a large dish antenna for signal reception, which has reportedly a diameter of ~100 m

Всякие секретные шаттлы на низкой орбите даже любители умудряются сфотографировать, а тут постоянно висящий в одной точке спутник. Есть же частные кампании, предоставляющие услуги (как я понял) в том числе фотографирования геостационарных спутников с Земли с целью мониторинга, есть огромное количество самых разных астрономических обсерваторий, есть спутники-шпионы на соприкасающихся орбитах, о которых постоянно журналисты с Пентагоном ноют. Почему бы не пустить ужа в американскую джинсовую штанину и не "слить" пару фотографий?

И из той же серии вопрос, почему нет характеристик орбит большинства "засекреченных" спутников? В чем беда условным китайцам скормить любителям орбиты, чтобы те расшарили их в свободном доступе, да еще фотографий наделали и посчитали, кто за кем и когда подслушивает? Китайцам терять нечего, их спутники американцы не секретят и любому желающему показывают.
Аноним 13/08/20 Чтв 10:57:46 595130340
>>594967
>Вояджер бы вылетел с одной стороны и прилетел к нам с противоположной, поскольку пространство замкнуто.
Пруфс?

>Но "между ними" ничего нет.
Так а откуда инфа-то эта? На основе чего ученые сделали такой вывод?
Аноним 13/08/20 Чтв 11:01:27 595131341
image.png 184Кб, 500x500
500x500
>>595112
Это радиоантенна, она не должна быть сплошной, потому ее хуево видно.
Аноним 13/08/20 Чтв 12:45:16 595137342
>>595112
Потому что эти йобы геостационарные, а значит слишком далеко. Геостационар это не то чтобы пиздец далеко, но объекты на таком расстоянии лишь как точки для лучшей оптики, деталей не различить. Секретный шаттл не летает к геостационарной, он на низкой орбите тусуется.
Аноним 13/08/20 Чтв 12:48:28 595138343
>>595112
>Почему бы не пустить ужа в американскую джинсовую штанину и не "слить" пару фотографий?
И что это даст? Амеры разноются, что наши спутники на геостационаре приближаются к ихним с вредными намеряниями опять, начнут вопить и получат дохуя денег для космических сил, установят на геостационарные спутники лазеры и будут палить без предупреждения в наши спутники при близком проходе.
Аноним 13/08/20 Чтв 13:45:06 595146344
>>595138
Нашние спутники не чета ихним!
Аноним 13/08/20 Чтв 14:42:13 595158345
>>595138
>установят на геостационарные спутники лазеры и будут палить без предупреждения в наши спутники при близком проходе
это же незаконно
Аноним 13/08/20 Чтв 15:10:20 595172346
>>595138
Ну вообще-то спутники там и так как в консервы понапиханы. И приближаться никуда не надо, просто поверни объектив и посмотри на спутник в 3х км от тебя. Правда не факт что это не шпионский спутник, а телекоммуникационный.
Аноним 13/08/20 Чтв 15:13:56 595175347
>>595172
>Ну вообще-то спутники там и так как в консервы понапиханы.
Йобу дал? Или у тебя мир барбисайз кербина?
400 спутников на 200 тысяч километров окружности.
Они, конечно, предпочитают ютиться над континентами, но их всего 400 штук, их совсем рядом не вешают.
Аноним 13/08/20 Чтв 16:57:56 595192348
>>595158
незаконно лишь размещение ядерного оружия в космосе, а обычное оружие можно размещать, его не размещают лишь на добром слове и потому что это непрактично, некрасиво и может спровоцировать визги и гонку вооружений. А может его размещают, но не трындят об этом потому что может спровоцировать визги и гонку вооружений.
Аноним 13/08/20 Чтв 19:39:56 595207349
>>595112
Потому что серьезные инструменты используют для другого, вояки свои фотки не показывают, а любители видят его на ГСО как точку.

Шаттлы и станция крутятся на НОО, которая в 100 раз ближе, несравнимые дистанции.

>почему нет характеристик орбит большинства "засекреченных" спутников?
Нет лишь характеристик американских спутников с секретными орбитами, и только в выгрузке NORAD. Так-то есть дохуя публичных или просто несекретных инструментов и сетей вроде того же пулкона, которые ведут иногда и их тоже, чтобы не столкнулись, так что принципиально публичная инфа есть. Вообще это довольно опасная хуйня, на амерских вояк бочку гонят уже много лет, что они орбиты секретят. Один хуй их вычислить не проблема, а управление спутниками затрудняет. С другой стороны, USSTRATCOM это единственный и огромный источник публичных данных по объектам на орбите, без них современные масштабы деятельности на орбите были бы затруднены. (а может наоборот, построили бы коммерческие сети НИПов и проблем бы не знали). Говорили вроде несколько лет назад про открытие ГККО СККП для публики, но вояки чёт уперлись.
Аноним 13/08/20 Чтв 21:52:43 595244350
У меня не вопрос а умеренно упоротый реквест.
Видео, снятое в космосе, ну очевидно с Мкс на котором была бы не "приколись Земля!!" и не "охуей мы тут болт пять часов вкручиваем" а звезды.
Наткнулся на одно такое с кометой и подохуел с чёткости звёзд.
Плеяды вообще как при идеальной атмосфере в бинокль.
Аноним 13/08/20 Чтв 21:56:13 595246351
>>595244
Даже 4к без зума не даст тебе нормальных звездю
МКС постоянно поворачивается и одну точку неба не увиишь
Аноним 13/08/20 Чтв 22:22:57 595252352
iss062e081621.jpg 3424Кб, 5568x3712
5568x3712
>>595244
Есть такие видео, но толку? Всё равно твой глаз совсем иначе увидит, чем камера-кодек-монитор, ты не ощутишь глубину как вживую.
https://www.youtube.com/watch?v=TnzyAZpE0a0
Аноним 13/08/20 Чтв 22:31:32 595254353
15972166571610.png 1081Кб, 1514x851
1514x851
Что можно сказать о мире этой планеты? На пике, если что, нарисован мир из мультсериала об аватаре Аанге ()и всратой Корре.
Аноним 13/08/20 Чтв 23:11:18 595267354
>>595254
Насколько я помню Аватар, там сайфай и летающие камни из анобтаниума, так что всерьез рассматривать его с точки зрения физики нелья.
Аноним 13/08/20 Чтв 23:30:05 595269355
Аноним 13/08/20 Чтв 23:31:17 595270356
>>595269
Не смотрел аниме по аватару, как и следующие фильмы смотреть не собираюсь.
Аноним 14/08/20 Птн 00:00:07 595274357
Аноним 14/08/20 Птн 00:16:39 595275358
>>595252
Мне нормально. Всё лучше чем из красной зоны.
Вот например, ну где и как я такое ещё увижу?
https://www.youtube.com/watch?v=Yh1_wHdUx3Y
Про этот канал я как раз знаю, неужели он один такой?
Аноним 14/08/20 Птн 00:18:42 595276359
>>595254
Геологически активный водно кислородный мир с крупным спутником в зоне обитаемости. Куда шапки на полюса проебали?
Аноним 14/08/20 Птн 22:54:28 595359360
image.png 823Кб, 1572x920
1572x920
Поясните плес почему водород в качестве рабочего тела для ЯРД выгоднее. Вот смотрю видос и тут эта табличка. И как бы да, скорость у частиц водорода больше, но импульс то больше у кислорода! Он весит в 16 раз больше, а значит и иvпульс будет в ~4 раза больше. Ну и еще жидкий водород ппц как тяжело хранить, а так вместо него можно взять хоть воду. Или нет?
https://www.youtube.com/watch?v=3aBOhC1c6m8
Аноним 14/08/20 Птн 22:57:45 595360361
>>595359
потому что удельный импульс, а не просто импульс, это разные понятия
Аноним 14/08/20 Птн 23:16:45 595364362
>>595359
Потому что в баки заливают не моли, а килограммы. Да, 1 молекула кислорода даст больший импульс, но мы же не молекулами отсчитываем.
Аноним 15/08/20 Суб 00:01:15 595372363
>>595360
А, понял. Для одной и той же самой массы мы выбросим x кг кислорода со скоростью 1944м/с, но с водородом для этих же х кг скорость будет 7737м/с.
ппц, даже в научпопе туплю
Аноним 15/08/20 Суб 00:07:33 595374364
>>595372
Знаешь, почему УИ в секундах?
"Сколько секунд сможет давать своей массе это топливо ускорение в 1g"
Аноним 15/08/20 Суб 14:24:12 595408365
1597490650835.jpg 1812Кб, 2560x1440
2560x1440
Поясните за СПУТНИКОВЫЙ ИНТЕРНЕТ.
Вот говорят, что Маск щас запустит спутники, раздаст интернет, и все будут сидеть через спутник и в хуй не дуть.

Меня же интересует, ЧТО ТАМ С ПРОПУСКНОЙ СПОСОБНОСТЬЮ.
Как какой-то милипиздрический спутник сможет обеспечить по 100 мегабит для каждого клиента?

На Земле зачем-то ебанистические датацентры строят, размером с завод и потребляющие гигаватты.

Аноним 15/08/20 Суб 14:27:50 595410366
>>595408
>Вот говорят, что Маск щас запустит спутники, раздаст интернет, и все будут сидеть через спутник и в хуй не дуть.
И похуй.
>Меня же интересует, ЧТО ТАМ С ПРОПУСКНОЙ СПОСОБНОСТЬЮ.
Современное железо нормально позволяет иметь гигабиты.
>Как какой-то милипиздрический спутник сможет обеспечить по 100 мегабит для каждого клиента?
А никак.
С таким пингом у тебя только стримминг нормальный будет, но не серфинг.
>На Земле зачем-то ебанистические датацентры строят, размером с завод и потребляющие гигаватты.
Датацентры ставят не для распределения траффика а для создания оного.
Распределяют траффик относительно небольшие штуки. В твоей квартире это коробочка размером с доширак, в твоем доме это коробочка размером с телевизор, в твоем районе это стойка размером с холодильник, в твоем мухосранске это помещение размером с хачмагаз.
Аноним 15/08/20 Суб 14:43:33 595416367
>>595410
> С таким пингом
Для спутника на высоте 500 км пинг туда и обратно 3 мс.
Больше уйдет на преобразования на спутнике и клиенте.

Люди сидят как-то на 4G с его 20-50 мс и им норм.

>В твоем мухосранске это помещение размером с хачмагаз
Серьезно? Есть фото?

И как это вообще технически должно работать?
Вот поймал спутник сигнал от абонента "хочу 2ch.hk". Далее он посылает сигнал на землю, там скачивают сайт, передают его обратно, а тот что делает?
Спутник не может направленно послать тому абоненту, что он хочел.
Сможет только тупо выдавать в общий эфир, а кому надо, тот поймает.

И вообще, тогда возникает проблема с фундаментальной пропускной способностью радиоканала, он не резиновый же.
Допустим, 10000 абонентов одного спутника решили одновременно посмотреть порно в 4к. При этом видео у каждого разное. Действия спутника?
Аноним 15/08/20 Суб 14:52:33 595417368
>>595416
>Для спутника на высоте 500 км пинг туда и обратно 3 мс.
>Больше уйдет на преобразования на спутнике и клиенте.
Посчитай их тоже.
>Люди сидят как-то на 4G с его 20-50 мс и им норм.
Окей. Я сидел на 150мс соединении. Это не то что я порекомендую, но для закачки это сносно.
>>В твоем мухосранске это помещение размером с хачмагаз
>Серьезно? Есть фото?
Их так много, что запрашивать это равноценно спрашивать как выглядит автомобиль. Загугли серверную твоего личного провайдера в твоем личном мухосранске. А можешь и не уточнять, т.к. по-хорошему внешне они различаются только тем как грамотно кабели уложены.
>И как это вообще технически должно работать?
>Вот поймал спутник сигнал от абонента "хочу 2ch.hk". Далее он посылает сигнал на землю, там скачивают сайт, передают его обратно, а тот что делает?
>Спутник не может направленно послать тому абоненту, что он хочел.
>Сможет только тупо выдавать в общий эфир, а кому надо, тот поймает.
Не понял сути вопроса, ты сам прекрасно все объяснил.
>И вообще, тогда возникает проблема с фундаментальной пропускной способностью радиоканала, он не резиновый же.
Дык. Потому никто всерьёз не воспринимает этот спам спутниками.
>Допустим, 10000 абонентов одного спутника решили одновременно посмотреть порно в 4к. При этом видео у каждого разное. Действия спутника?
Обосрётся.
Аноним 15/08/20 Суб 15:19:19 595419369
>>595417
>Дык. Потому никто всерьёз не воспринимает этот спам спутниками.
Опять усталость металла.
Спам спутниками воспринимают серьезно не потому что у каждого жителя города будет вместо его дом.сру спутник, а потому что спутник будет у тех до кого провы в принципе добираться не будут. У для роботизированной агротехники и тп.
Аноним 15/08/20 Суб 15:24:25 595420370
>>595419
Ты не с тем споришь. Не туда воюешь. Я знаю как применять такое, я отвечал анону его словами.
Аноним 15/08/20 Суб 15:36:16 595421371
>>595420
>Ты не с тем споришь. Не туда воюешь. Я знаю как применять такое, я отвечал анону его словами.
У тебя получилось эталонное "старлинк ненужен" же.
Аноним 15/08/20 Суб 15:44:11 595423372
>>595421
Ты увидел это, но это не был моим посылом, я отвечал на его вопросы. ЕМУ и МНЕ старлинк реально не нужен. Вообще он может иметь применение, с этим никто не спорит.
Аноним 15/08/20 Суб 16:39:32 595433373
>>595419
Какая нахуй агротехника, нахуя ей спутник? Мобильного инета недостаточно?
Аноним 15/08/20 Суб 17:19:19 595434374
>>595267
>>595276
Нет, я имел в виду сопоставление с размерами планетиы, ибо в Аватаре она какая-то уж слишком мелкая. Помню, в Аанге в 3 сезоне они весь мир от поюсадо полюса долетели за какие-то считанные дни. Она же не может быть с Луну земную
Аноним 15/08/20 Суб 17:50:25 595436375
>>595433
Агротехнике в антарктиде или в горах, где нет мобильного интернета.
Аноним 15/08/20 Суб 17:58:12 595438376
>>595434
Нормальный, среднестатистический, дозвуковой, пассажирский самолёт долетит от полюса до полюса за сутки без пересадки. Я конечно дальше 1 сезона не смотрел, но та летающая бабайка 200км\ч точно сможет. Если она будет лететь по 12 часов в сутки, то 20 000 км это 8 дней. А если я правильно помню то в той серии эта бабайка косплеила дальнобойщика и не спал, так что ополовинивай, до 4 дней. Это при условии, что 200 км\ч предел, в чём я сомневаюсь.
Аноним 15/08/20 Суб 17:58:30 595439377
Аноним 15/08/20 Суб 18:34:18 595441378
>>595433
>>595436
Дети бетонных джунглей, которые никогда не были в поле
Аноним 15/08/20 Суб 18:47:06 595443379
>>595438
Та серия в середине второго сезона была. В конце сериала аанг пропал и шайка его, типа, за неделю с копейками весь мир облетели в его поисках, а так его и не нашли. Так что там не прямая от полюса до полюса, а галопом по европам с поворатми в разные города и деревни
Аноним 15/08/20 Суб 19:07:28 595446380
>>595443
Всё же зависит от скорости. Ничто не мешает этой ебабаки двигатся 800 км\ч и за неделю осуществить 3,3 оборота вокруг.
Аноним 15/08/20 Суб 19:13:25 595448381
>>595441
Ничего не знаю житель усть-залупинска. Во всех полях где я был было 4ж.
Аноним 15/08/20 Суб 19:15:24 595449382
>>595438
>>595446
Они же летали верхом на этой штуке их бы сдуло воздухом нахуй на таких скоростях. Думаю, ее средняя скорость киллометров 60 в час.
хотя на них летают маги воздуха, мб они как-то компенсируют это сопротивление
Аноним 15/08/20 Суб 19:23:27 595450383
>>595438
Так то современные пассажирские самолёты по скорости ебут в рот истребители времен войны, например.
Аноним 15/08/20 Суб 20:13:42 595454384
>>595448
Это мелкие. Бывают такие поля что за горизонт уходят, там никаких вышек не поставишь. Бывают на уклонах и т.п.
Аноним 15/08/20 Суб 20:20:39 595458385
>>595454
Обосрался с поля от горизонта до горизонта. И вообще, зачем на поле 4г? Чтобы сидеть в тиктоке, пока на комбайне хуячишь? Я не уверен, что селюки вообще знают, что такое интернет.
Аноним 15/08/20 Суб 20:34:09 595460386
>>595458
>Обосрался с поля от горизонта до горизонта.
Это пока не увидел.
>И вообще, зачем на поле 4г?
Анон же говорил, >для роботизированной агротехники
Аноним 15/08/20 Суб 20:34:23 595461387
>>595458
Про автоматические комбайны не слышал что-ли ?
Аноним 15/08/20 Суб 20:36:27 595462388
Аноним 15/08/20 Суб 21:30:44 595474389
>>595462
Селюк тебе сможет 24/7 пахать?
Аноним 15/08/20 Суб 21:33:38 595478390
>>595474
>Селюк тебе сможет 24/7 пахать?

Ну там даже при наличии робота все равно будет контроль оператора скорее всего - а значит тоже не 24/7. Но в принципе наверное можно одного оператора на несколько комбайнов.
Аноним 15/08/20 Суб 21:34:21 595479391
>>595474
А тебе прямо обязательно 24/7?
Аноним 15/08/20 Суб 21:40:35 595480392
>>595479
Не совсем, но смотри:
- селюку нужно платить зарплату
- селюк может разъебать технику
- селюк может просто забухать и не выйти
- селюка не получится вытащить в его нерабочие дни
и так далее. С беспилотным тебе из людей нужен будет оператор, который отправит задание на день и периодически мониторить на случай внештатных ситуаций и общее состояние техники. Причём вообще похер из какой точки земного шара.
Аноним 15/08/20 Суб 21:42:30 595481393
>>595480
Осталось изобрести самообслуживающийся комбайн.
Аноним 15/08/20 Суб 21:47:25 595482394
>>595480
>Причём вообще похер из какой точки земного шара
Колесо отвалилось из-за бракованных болтов. Ты полетишь из другой точки земного шара?
Аноним 15/08/20 Суб 21:48:42 595484395
>>595482
Я нажму на кнопку, и благодаря мускоинтернету комбайн получит команду прикрутить это колесо!
Аноним 15/08/20 Суб 21:49:29 595485396
>>595484
Очень здорово. А прикручивать колесо-то кто будет?
Аноним 15/08/20 Суб 21:52:52 595486397
>>595485
Селюков через интернет наймём.
Аноним 15/08/20 Суб 21:54:47 595487398
Аноним 15/08/20 Суб 21:55:37 595488399
>>595450
>>595449
Мне думается 200 точно должно быть. Мотоциклистов то не сдувает. А 60 там вообще почти не чувствуешь, если в одежде.
Аноним 15/08/20 Суб 21:59:23 595489400
>>595482
>Колесо отвалилось из-за бракованных болтов. Ты полетишь из другой точки земного шара?
Ну для самолетов буквально так и делают.

По статистике из статьи выше 30% времени что ли комбайнер проводит за ремонтом. Это конечно много но уже значит что вместо 3х комбайнеров на поле нужен будет 1 ремонтник.
Аноним 15/08/20 Суб 22:01:59 595491401
>>595488
Немцы вообще срать умудрялись, высунув жопу из штуки, а она до 300 в горизонте могла разогнаться при желании, если верить википедии.
Аноним 15/08/20 Суб 22:08:06 595493402
>>595489
Так зачем в итоге интернет, если за все операции может отвечать тот же оператор-ремонтник-селюк? И потребуется не 100500 в месяц, а один раз купить рацию за 1000 деревянных.
Аноним 15/08/20 Суб 22:21:35 595494403
>>595491
У них что там, по двенадцать часов вылеты были? Можно ж как-то рассчитывать время для сранья, чтобы срать с комфортом в туалете типа сортир.
Аноним 16/08/20 Вск 00:11:54 595517404
Здравствуйте, это канал об агротехнике? Как пропатчить PolyCAN под SAE J1939 2.1?
Аноним 16/08/20 Вск 06:46:22 595545405
>>595512
>Пилот не ремонтирует самолет, водятель не ремонтирует авто. То что агротехника исключение из этого правила, не значит что это исключение скоро не закончится потому что условия изменятся.
Ебать, автомобиль и тем более самолёт это по сущности своей быстроходный транспорт, заточенный на преодоление больших расстояний. Хотя и автомобили без "ближайшего мухосранска" встанут раком, ибо в соседний город до сервиса не поедешь, если у тебя маслонасос полетел.

А с комбайном как? Своим ходом через всю страну хуярить на завод-изготовитель? Ан-124 вызывать? Сельхозтехника это почти недвижимость. Ты, конечно, можешь попытаться построить в чистом поле полностью автоматизированный ремонтный цех, если дохуя человеконенавистник, но в любой непонятной ситуации мотаться до столицы комбайн не будет. А выездной мастер в чистом поле его не отремонтирует. Да и жить ему нужно не в палатке, а ремонт может затянуться и на несколько дней. И так постепенно приходим к тому, что один хуй нужна автономная база, а вокруг базы автономный городок, потому что это минимальная автономная единица человеческой цивилизации.

А так, конечно, голубая мечта всякого маняурбаниста - согнать всё население в Гигахрущ, приковать к терминалам, а земные пустоши заполнить зондами фон Неймана. Это ж сколько денег сэкономится, пиздец, можно три яхты купить.
Аноним 16/08/20 Вск 08:45:19 595549406
>>594564
>Водой - нет, она испарится.
Если только не замерзнет. Причем при ниже -200С лед не сублимируется.
Аноним 16/08/20 Вск 08:49:21 595550407
0002-002-Postoj[...].jpg 65Кб, 960x720
960x720
>>594937
>магнитное поле не существует само по себе, а только порождается электрическим током
ORLY?
Аноним 16/08/20 Вск 08:57:40 595551408
>>595408
Скорость света в волокне сильно меньше скорости света в вакууме.
Аноним 16/08/20 Вск 09:00:03 595552409
>>595417
>Я сидел на 150мс соединении
Я и сейчас сижу, и даже танки катаю. Вот больше уже некомфортно.
Аноним 16/08/20 Вск 12:48:16 595566410
>>595550
Даун, почитай, откуда магнитный момент в веществе берётся — из тех же замкнутых электрических токов.
Аноним 16/08/20 Вск 14:50:15 595576411
>>595549
У воды огромная теплоемкость, а в космосе очень херовая теплоотдача, практически только излучением. Не замерзнет.
Аноним 16/08/20 Вск 15:14:17 595581412
Аноним 16/08/20 Вск 15:25:35 595584413
Почему говорят, что пилоты при полетах на сверхзвуке не слышат звука моторов, обгоняют его? Но космонавтам это же не мешает слышать. Даже в космосе слышат. Звук же через корпус передается с куда большей скоростью. Звучание звука по своим характеристикам совершенно другое будет? Или это байка для колхозников?
Аноним 16/08/20 Вск 15:40:23 595586414
>01:05 если лопнуло остекление голову убираешь в воздушный карман, он всё-равно будет сверху
https://youtu.be/LlqkuSfxiKw?t=63

КАК? Скафандр такой огромный, что в нём можно запрокинуть голову носом вверх?
Аноним 16/08/20 Вск 16:47:37 595600415
>>595581
Электрон не шарик, но орбитальный момент, спин и возникающие в их результате магнитные поля у него такие же, как если бы он был шариком. Он не обладает врожденным магнитным полем, магнитный момент у него возникает именно из-за заряда, "движения" и "вращения" (в кавычках).

https://en.wikipedia.org/wiki/Magnetic_moment#Atoms,_molecules,_and_elementary_particles
Аноним 16/08/20 Вск 16:53:08 595605416
>>595581
>У тебя небось и электрон это маленький шарик, летающий вокруг ядра?
Это как бы равновероятностное облако маленьких шариков, равновероятностно обнаруживающихся летаемыми вокруг ядра. С точки зрения магнитного поля это совершенно похуй.
Аноним 16/08/20 Вск 16:54:35 595606417
>>595584
Звук двигателя в основном производится не в камере сгорания и не от вибраций мотора, а от ударных волн в выхлопе и турбулентностей при смешивании его с окружающим воздухом. А как только выхлоп покидает сопло двигателя, он уже полностью расцеплен с корпусом и шум до кабины пилотов никак долететь не может.
Аноним 16/08/20 Вск 17:28:30 595619418
Какой пинг с МКС, можно ли играть в мультиплеер если хочется?
Аноним 16/08/20 Вск 17:37:19 595620419
Delta IIDamage.jpg 172Кб, 1200x800
1200x800
Пасаны, у меня тут после трёхдневного отключения интернета внешними врагами™ в Беларуси перестал быть доступным офсайт NASA и некоторые другие их сайты, ну и за компанию nextspaceflight. Остальные сайты из США работают, в том числе государственные. Чо делоть-то?
Сижу через 4G момед, без прокси, опсос A1, браузер лiсiчка.
Аноним 16/08/20 Вск 17:40:20 595621420
>>595620
>Чо делоть-то?
ВПН подрубать, ебать провайдера
Аноним 16/08/20 Вск 17:42:29 595622421
>>595620
Ещё несколько дней, и наведут порядок и интернет снова будет. А пока сиди тихо.
Аноним 16/08/20 Вск 17:45:17 595623422
>>595622
Интернет уже есть, долбоёб.
Аноним 16/08/20 Вск 17:58:50 595624423
>>595620
ВПН, что еще можно сделать-то.
Аноним 16/08/20 Вск 21:20:14 595642424
Может ли ТМ взаимодействовать только гравитацией?
Могут ли частицы никак не взаимодействовать в принципе?
Аноним 16/08/20 Вск 21:31:53 595643425
>>595619
Хуевый, нельзя. То что там есть интерактивного, идет в интернет через спутники TDRS и один из центров насы, иногда огибая землю до другого конца..
Аноним 16/08/20 Вск 21:33:41 595645426
>>595586
Головой там свободно крутится, это очевидно даже глядя снаружи.
Аноним 16/08/20 Вск 22:07:44 595663427
>>595642
>Может ли ТМ взаимодействовать только гравитацией
Может. А может и взаимодействовать с другой ТМ.
>Могут ли частицы никак не взаимодействовать в принципе?
Могут. Но ты их никак не найдёшь
Аноним 16/08/20 Вск 22:13:19 595665428
>>595663
> не найдёшь
Ну можно например увидеть отсутствие эмиссионного следа. Частица А взаимодействует и порождает частицу Б. А взаимодействует, Б нет. Не?
Аноним 16/08/20 Вск 22:16:07 595667429
>>591100 (OP)
В этом вашем Сатурне-1 как первая ступень устроена?
Аноним 17/08/20 Пнд 00:27:02 595680430
BlueprintSA-5 S[...].jpg 857Кб, 2298x2952
2298x2952
BlueprintSA-5 S[...].jpg 1199Кб, 2346x3132
2346x3132
BlueprintSA-5 S[...].jpg 2100Кб, 2298x3048
2298x3048
BlueprintSA-5 S[...].jpg 1791Кб, 2262x2772
2262x2772
Аноним 17/08/20 Пнд 00:36:30 595681431
>>595680
Короче, чисто А5 с возвращаемым блоком I-II ступени, лел.
мимокрок
Аноним 17/08/20 Пнд 00:41:02 595683432
>>595680
Что такое ANTISLOSH BAFFLES и почему этого нет насколько мне известно в остальных ракетах?
Аноним 17/08/20 Пнд 00:43:05 595684433
>>595683
Это перегородки, мешающие жидкости плескаться как хер в рукомойнике. На других ракетах они есть, в т. ч. верхних ступенях сатурнов. И на бензовозах
Аноним 17/08/20 Пнд 00:48:53 595686434
Аноним 17/08/20 Пнд 00:55:47 595687435
>>594571
Не так. Разрешение в центральной ямке довольно хоршее и так. Но за счёт хитрого алгоритма обработки из нескольких кадров составляется изображение ещё лучше, чем просто принтскрин в один момент времени. Кто проверялся у офтальмолога, знает, что зрение двумя глазами чётче, чем одним.
Алсо, недавно или давно обкатывали технологию, позволяющую восстанавливать более чёткое изображение при нескольких сканирований матрицы. Принцип тот же.
Аноним 17/08/20 Пнд 01:21:32 595688436
>>595687
я так примерно почувствовал, что с сетчатки глаза не должно быть покадрого съёма данных как в фотиках, но каждая коблочка палочка шлёт данные независимо и паралельно прямо в мозг по мере поступления фотонов, я не прав?
Т.е. это не срезы информации там пусто / тут густо
а как бы фонтанчики в лейке душа,где каждая струйка "плавно" меняет напор от нуля до максимума и мозг имеет как бу меняющийся под пальцами ландшафт, который он в реалтйаме ощупывает и делает выводы
Аноним 17/08/20 Пнд 01:24:40 595690437
>>595688
В общем, она не отправляет в мозг дискретные кадры, а срёт непрерывным аналоговым сигналом. Поэтому я и писал скриншот"
Аноним 17/08/20 Пнд 01:47:20 595695438
Где-то слышал, что глаз воспринимает детали лучше, чем во много раз большее по диаметру зеркало снятое на фото. Можно ли как-то улучшить астро-фото за счёт молекулярной матрицы подобной глазу?
Аноним 17/08/20 Пнд 13:40:13 595742439
>>595695
Не слушай там больше. Детали ограничены размером оптической системы. https://ru.wikipedia.org/wiki/Диск_Эйри
Глаз - точно такая же оптическая система, с точно такими же чувствительными элементами.

>>594571
>>595687
>>595688
>>595690
>В общем, она не отправляет в мозг дискретные кадры, а срёт непрерывным аналоговым сигналом
Датчики тоже такие бывают, асинхронный дифференциальный сигнал это в последние пару лет горячая тема для всяких служебных камер, где на том конце не человек, наблюдающий красивую картинку, а система машинного зрения. Позволяет сильно сократить энергопотребление и требуемую пропускную способность системы.
Аноним 17/08/20 Пнд 13:41:57 595743440
>>595684
Даже на молоковозах. Везде где жидкости возят.
Аноним 17/08/20 Пнд 13:48:33 595744441
>>595643
Могут ли космонавты капчевать с МКС?
Аноним 17/08/20 Пнд 13:50:46 595745442
>>595744
Изначально не могли, им по запросу странички качали и кэшировали. Но с некоторых пор могут и напрямую.
Аноним 17/08/20 Пнд 14:06:57 595749443
>>595745
А были случаи когда космонавты на дваче или форчане с пруфами с МКС сидели?
Аноним 17/08/20 Пнд 14:07:17 595750444
>>595745
Не напрямую. А то вишмастер качнут и МКС зависнет. Они по удаленке с пекарней в Хьюстоне соединяются, а вот она уже доступ в инет имеет.
Аноним 17/08/20 Пнд 14:08:04 595751445
Аноним 17/08/20 Пнд 14:15:06 595753446
>>595751
На реддите не интересно, реддит превратился в рафинированную помойку после того как китайцы его выкупили. Меня интересуют случаи типа продырявлю обшивку МКС на квадрипл.
Аноним 17/08/20 Пнд 14:16:44 595754447
>>595753
Космонавты проходят психологический отбор. На орбиту питурдов не берут.
Аноним 17/08/20 Пнд 14:29:59 595757448
>>595750
Не, именно что напрямую. Надо искать архитектуру системы, я помню что там есть тухлый, но прямой линк через TDRSS, и впоследствии через прокси в одном из центров насы. А вишмастеров на айпадах нет, да и выделены они под подобную слабокритичную хернь.
Аноним 17/08/20 Пнд 14:32:34 595758449
>>595743
Можно ли создать ракету на молоке для взлета хотя бы с километрового астероида?
Аноним 17/08/20 Пнд 14:34:24 595759450
>>595758
С километрового можно хоть на моче взлететь.
Аноним 17/08/20 Пнд 14:34:39 595760451
>>595757
>TDRS
Я одно время думал что они по спутникам Луч в интернеты выходят. Вроде аж сам Наше Всё про них говорил.
Аноним 17/08/20 Пнд 14:37:18 595761452
>>595760
Луч заработал недавно, а используют интернет (напрямую или по запросу) с самого начала US-сегмента.
Аноним 17/08/20 Пнд 14:38:30 595762453
>>595759
Можно ли создать ракету на молоке для взлета хотя бы с Деймоса?
Аноним 17/08/20 Пнд 14:39:18 595763454
>>595762
Да даже с Фобоса.
Аноним 17/08/20 Пнд 15:00:43 595765455
Аноним 17/08/20 Пнд 15:03:11 595766456
>>595765
Там можно жопой на криовулкан сесть и взлететь
Аноним 17/08/20 Пнд 15:03:45 595767457
>>595766
В чем принципиальная разница у кривовулканов и вулканов здорового человека?
Аноним 17/08/20 Пнд 15:06:15 595768458
>>595767
в температуре и составе
криовулканы это просто вода, иногда даже не горячая для человека
Аноним 17/08/20 Пнд 16:38:37 595787459
Мужики, а как там Луна влияет на моря?
Аноним 17/08/20 Пнд 16:39:20 595788460
>>595787
Притягивает воду создавая приливы.
Аноним 17/08/20 Пнд 16:39:20 595789461
Аноним 17/08/20 Пнд 16:58:52 595792462
Аноним 17/08/20 Пнд 17:18:25 595795463
Можно ли создать матрицу, которая будет регистрировать каждый фотон попадающий на неё?
Аноним 17/08/20 Пнд 17:21:04 595796464
>>595795
И да, и нет. В обычном понимании как фотокамера но для любого фотона нельзя. Как научный инструмент работающий на всех длинах волн - такое реально сделать, но оно не будет напоминать фотоматрицу никак, это будет хтоническая йоба.
Аноним 17/08/20 Пнд 17:35:01 595797465
>>595766
хуй вот, там говорят криовулканы извергаются как на комете конча выделяется. Возможно будешь стоять на поверхности и не будешь в упор видеть криовулканы, лишь какие-то трещины закрытые лёгким туманом и дымкой до небес. Как извержение оно видимо лишь с больших расстояний и при выгодном освещении.
Аноним 17/08/20 Пнд 17:39:01 595798466
>>595797
Jets moving 250 kg of water vapor every second at up to 2,189 km/h into space
Аноним 17/08/20 Пнд 17:39:47 595800467
Аноним 17/08/20 Пнд 17:43:57 595804468
>>595800
>Enceladus expels around 250 kg of water vapour every second, through a collection of jets from the south polar region known as the Tiger Stripes because of their distinctive surface markings.
да, наверное все таки на плюме не улетишь никуда
Аноним 17/08/20 Пнд 17:47:00 595805469
>>595804
А если собрать большой форсункой?
Аноним 17/08/20 Пнд 17:49:30 595806470
>>595805
Не проще ли собрать нормальный летательный аппарат тогда уж?
Аноним 17/08/20 Пнд 17:50:34 595807471
Там может иней всё покрывает и аморфный снег выпадает, его можно собирать, разогревать и на паровом двигателе лететь.
Аноним 17/08/20 Пнд 17:52:42 595808472
>>595806
Ну, я предлагал ракету на молоке, а мне про криовулканы начали затирать.
Аноним 17/08/20 Пнд 17:57:00 595809473
>>595808
Я скажу одну простую вещь, ты только не обижайся. Молоко - весьма плохое топливо для ракет. Даже в холловском движке не айс, будет забивать форсунки свернувшимися белками.
Но если про такое рассуждать, то молочная ракета это наверное валидный способ взлета с планеты состоящей из моля кротов. https://chtoes.li/a-mole-of-moles/
Аноним 17/08/20 Пнд 19:04:07 595813474
>>595796
Я имел ввиду все фотоны определённой длинны волны от и до. В контексте эффективности матриц телескопов.
Аноним 17/08/20 Пнд 19:05:27 595814475
>>595795
Можно. Основных препятствий несколько:
- в силу квантовой механики посчитать один фотон трудно, у тебя получится неопределенность/шум https://en.wikipedia.org/wiki/Shot_noise#Optics
- электрически усилить одно очень мелкое событие очень трудно. Но возможно при помощи определенных типов лавинных фотодиодов (APD, юзабельных в матрице камеры), они приближаются к однофотонной регистрации, жертвуя другими параметрами.
https://en.wikipedia.org/wiki/Single-photon_avalanche_diode
А также при помощи каскадных фотоэлектронных умножителей на базе ламп (как это ни странно), это уже не единая полупроводниковая матрица.
https://en.wikipedia.org/wiki/Photomultiplier_tube
Такие могут использоваться например в нейтринных телескопах, где "пиксели" разнесены в пространстве, однако ничто не мешает там и SPAD/полупроводниковые детекторы использовать.

Все большие оптические телескопы в наше время оперируют отдельными фотонами, по сути, и ограничены лишь своей оптикой, а не матрицей детектора. Поэтому кстати и были претензии к старлинку, и остались после покраски. Невозможно вычесть их из сигнала, т.к. даже если ты знаешь что за ту долю секунды пролёта спутника по кадру ты получишь ровно 100 фотонов, на самом деле по распределению Пуассона ты получишь (условно говоря) от 90 до 100. Вуаля, ты только что получил неопределенность в 20 фотонов, которое делает всю выдержку бессмысленной, потому что от супер-тусклого объекта, который ты наблюдаешь, может прилететь например 2 фотона за всю выдержку в несколько часов. А именно такие объекты обычно и интересны.
Аноним 17/08/20 Пнд 19:06:01 595815476
>>595813
Нет, 100% фотонов не поймать. Часть будет поглощена оптической/направляющей системой, часть не провзаимодействует с матрицей.
Аноним 17/08/20 Пнд 19:07:01 595816477
>>595789
Почему тогда с противоположной от Луны стороны тоже прилив?
Аноним 17/08/20 Пнд 19:36:10 595819478
>>595816
Потому что там притяжение Луны слабее того, что действует на центр Земли.
Аноним 17/08/20 Пнд 19:38:37 595821479
image.png 436Кб, 1247x673
1247x673
Аноним 17/08/20 Пнд 22:20:51 595835480
>>595789
скорее не притягивает, а сплющивает с боков.
Аноним 17/08/20 Пнд 22:21:36 595836481
Аноним 17/08/20 Пнд 23:09:59 595838482
>>595819
>>595821
С противоположной стороны от Луны прилив меньше? если да, то насколько?
Аноним 17/08/20 Пнд 23:17:18 595839483
>>595838
Да, меньше, но разница там пиздец маленькая. Что-то типа (1/60^2 - 1/61^2) / (1/59^2 - 1/60^2) ~= 95,1%, если не ошибаюсь (60 - примерное расстояние в радиусах Земли). Короче, на 5% меньше должны получаться.
Аноним 18/08/20 Втр 02:23:30 595852484
У нас есть частица не нулевой массы, двигающаяся с релятивистской скоростью. Из за лоуренцева сокращения земля будет плоским блином. Правильно ли я понимаю, что при достаточной скорости частица может квантово туннелировать сквозь всю землю с достаточно высокой вероятностью?
Аноним 18/08/20 Втр 02:41:22 595855485
1306301474810.jpg 27Кб, 600x337
600x337
>>595665
>увидеть отсутствие эмиссионного следа
Эдак можно сразу много открытий понаделать.
Аноним 18/08/20 Втр 02:48:31 595856486
>>595855
Тих ты, не мешай, мы щас так штук шесть фундаментальных взаимодействий откроем, которые с обычной материей не реагируют.
Аноним 18/08/20 Втр 03:11:33 595860487
>>595855
Имел ввиду что баланс энергии в эксперементе будет смещён. Например в сумме в детекторе 3 ТЭВ, а вылетело 2.99 ТЭВ. В итоге куда-то делось 10 ГЭВ. Одно из объяснений невзаимодействующая частица.
Аноним 18/08/20 Втр 03:12:42 595862488
Аноним 18/08/20 Втр 03:15:00 595863489
image.png 221Кб, 450x424
450x424
>>595862
Овощную тему закрыли, открыли кисломолочную?
То анон на молоке ракету, то ты поминаешь всуе.

Кстати, а если скормить бактериям то молоко на котором анон летать собрался, они же напукают углекислого газа и дадут больше давления?
Аноним 18/08/20 Втр 14:28:56 595909490
А возможно в теории построить ебический циклотрон и гонять там тяжелые элементы на релятивистских скоростях, чтоб несмотря на миллисекунды полураспада из-за замедления времени мы их могли годами хранить?
Аноним 18/08/20 Втр 15:26:26 595918491
>>595863
>Lactobacillus acidophilus
О, надо ебнуть ацидофилинчика, как раз в холодильнике открытый стоит.
Аноним 18/08/20 Втр 15:27:37 595919492
yoba cosmonauts.jpg 197Кб, 1024x682
1024x682
>>595918
Спейсач оздоровляющий.
Аноним 18/08/20 Втр 15:28:48 595920493
Есть новости по скафандрам для ЕВА, они такие же громоздкие как и раньше, или есть подвижки в разработке?
Аноним 18/08/20 Втр 15:35:46 595921494
>>595919
Космонаут на заднем плане не задохнется в вакууме? Йобы-то вон в скафандрах.
Аноним 18/08/20 Втр 15:37:35 595922495
>>595921
Так это женщина же. Они так аборты на МКС проводят.
Аноним 18/08/20 Втр 16:17:02 595927496
>>595920
Никаких практически. Компрессионные как были мемом, так и остались.
Аноним 18/08/20 Втр 16:17:45 595928497
Бывают ли урановые астероиды?
Аноним 18/08/20 Втр 16:19:22 595930498
>>595909
Нужно ОСНЕ дохуя энергии и размеры ебические. Но зачем их хранить, когда можно генерировать.
Аноним 18/08/20 Втр 16:22:51 595931499
>>595928
Не бывают, уран поймался в планеты при формировании солнечной системы.
Аноним 18/08/20 Втр 16:29:05 595933500
>>595930
Чтобы успеть с ними эксперименты провести.
Например, остров стабильности слишком слабый и какой-нибудь оганессон не хранится. А что если мы два оганессона склеим, например? Вдруг там дальше есть остров стабильности и получится генерить элемент 200 какой-нибудь который не будет распадаться и будет ебически плотным источником энергии?
Аноним 18/08/20 Втр 16:47:07 595937501
>>595933
Подозреваю что время слияния двух 100 протонных фермиев будет считаться по их времени, а не по нашему. И они будут сливаться 100 тысяч лет по нашему времени, а распадаться за 3 года. Так же подозреваю, что они при столкновении скорее разобьются на 4 более мелких ядра, чем сольются в одно большое.
Аноним 18/08/20 Втр 17:20:05 595942502
>>595933
Это и за долю наносек делается отлично, особенно когда у тебя непрерывно хуячит пучками
Аноним 18/08/20 Втр 17:31:59 595957503
>>595942
У тебя нет 120 элемента. Ты его должен получить сперва.
А потом его же столкнуть с другим таким же.
Ты не успеешь, так не делают пока что.
Аноним 18/08/20 Втр 21:17:24 596027504
https://ru.m.wikipedia.org/wiki/Токамак
А мне тут говорили, что эффективные двигатели с очень горячим рабочим делом не построишь.
Что мешает построить корабль с таким токамаком, нагреть в нем водород до 100500 мильйонов кельвинов и высирать его из сопла со скоростью в 99% света?
Алсо, какой удельный импульс у такой йобы будет?
Аноним 18/08/20 Втр 21:19:42 596031505
>>596027
Мешает то, что всё нагреется и расплавится. На земляшке почему электростанции возле рек стоят или градирни имеют? Чтобы иметь теплообмен.
В космосе ты не сможешь запросто температуру сбрасывать.
Аноним 18/08/20 Втр 21:21:54 596032506
>>596031
Так токамак и нужен, чтобы этот плазменный водород не касался стенок двигателя, тем самым, не нагревая корабль.
Аноним 18/08/20 Втр 21:24:36 596033507
image.png 170Кб, 512x215
512x215
>>596032
Ты забыл про бульон излучение, у токамаков стенки охлаждаются активным образом.
Без охлаждения стенки поплавятся, магниты поломаются и плазма убежит.
Куча токамаков вообще на сверхпроводящих магнитах работают и требуют -200С температуры примерно. А внутри 1000000000С
Так дела не работают.
Аноним 18/08/20 Втр 21:40:09 596035508
image.png 72Кб, 1488x211
1488x211
>>596033
Эххх, жаль :(
Мб в будущем токамацкие технологии улучшат, что для охлаждения достаточно будет и радиаторов в вакууме.
Аноним 18/08/20 Втр 21:53:46 596038509
можно ли ходить по марсу только в шлеме с подачей нужного воздуха, и в обычной зимней одежде? чтобы срать на марес не снимая скафандр
насолько я понимаю, у тонкоатмосферных планет беда с башкой радиацией. но если эта проблема оказывается решина (не знаю как, просто берём, что эта проблема решина), то всё будет норм?
Аноним 18/08/20 Втр 22:04:01 596041510
>>596035
Анончик, дружиже, дело в термодинамике. Тут нечего улучшать. Нам надо запилить системы охлаждения. В безвоздушном пространстве космоса это огромные простыни радиаторов или капельные холодильники.
Сможем в охлаждение - сможем говорить о более тепловыделяющих энергетических установках.
Аноним 18/08/20 Втр 22:04:19 596042511
>>596038
Там на поверхности давление атмосферы ~6 мБар. Это конечно, не совсем вакуум, но охренеть как мало (на Эвересте ~300 мБар) и сосуды будет изнутри разрывать давлением крови.
Аноним 18/08/20 Втр 22:06:42 596043512
>>596038
Нет, нельзя.
Тебе когда-нибудь делали засос?
Не переживай, сычинушка-корзиночка, ты можешь его себе сам сделать, присосись к руке и соси. Ты создашь разницу давлений грубо говоря в половину атмосферы. Теперь представь, что сосёт в 100 раз мощнее. И этот засос на всём твоём теле.
Поэтому нужен скафандр, потому, что тебе будет сосать из ануса и пениса, тело покроется повреждениями из-за отсутствия давления.
Ты можешь выжить пару минут в экстренном режиме.
Жить нормально ты не сможешь даже близко.
Аноним 18/08/20 Втр 23:10:53 596065513
>>596038
Почитай про альпинизм, например. Нубы (да и все кроме хардкорщиков) залазят на всякие восьмитысячники с дополнительным кислородом. Вообще, все что выше 8000 называется "зоной смерти", пушо человек там жить может, но не долго. Даже с искусственным кислородным наддувом. А там, между прочим, давление около 1/3 атмосферы - и даже этого уже мало.
Аноним 18/08/20 Втр 23:16:18 596066514
>>596065
Та достаточно там давления. А вот кислорода не хватает. В аполлонах вроде было давление в треть атмосферы, но космонавты дышали чистым кислородом. И ниче, неделю нормально прожили, даже на луне побывали.
Аноним 18/08/20 Втр 23:18:29 596067515
image.png 3996Кб, 1424x1068
1424x1068
>>596066
А еще тройка удачно так пропеклась.
Аноним 18/08/20 Втр 23:19:17 596068516
Возможно ли сделать прямоточный реактивный двигатель за счёт толкания виртуальных частиц сильным полем?

Допустим делаем полу-бублик "обрезками" назад.
виртуальные пары электрон/позитрон разделям сильным электромагнитным полем и по дуге выплёвываем назад, электроны из одной трубы, позитроны из другой всё назад.
Т.е. получается реактивная тяга на таком типа искусственном излучении Хокинга, причём рабочее тело рождается из "кипящего вакуума".
Аноним 18/08/20 Втр 23:19:50 596069517
>>596066
Ну хз, значит там наддув слабоватый. В конце концов, им же надо это на своем (или шерповском) горбу еще наверх тянуть, плюс климат не тот.
Аноним 18/08/20 Втр 23:21:42 596070518
>>596069
Ты должен понимать разницу в парциальном и абсолютном давлениях.
Чистый кислород подавался в почти что вдвое большем давлении чем при нормальной атмосфере.
И его не юзают в космосе сейчас несмотря на выгоду в массе потому, что сг... Потому, что вот >>596067
Аноним 18/08/20 Втр 23:22:39 596071519
>>596068
>Возможно ли сделать прямоточный реактивный двигатель за счёт толкания виртуальных частиц сильным полем?
Нет.
Ты начал думать про этот вопрос. Попробуй сам подумать и опровергнуть свою гипотезу, это научный подход. Сможешь?
Аноним 18/08/20 Втр 23:25:58 596072520
>>596070
Не, я понимаю, понятно что на Эвересте атмосферного кислорода мало. Но ведь и дополнительный не спасает, неделю там все равно не прокукуешь. Наверное, просто, дополнительного из стандартных баллонов мало.
Аноним 18/08/20 Втр 23:32:12 596073521
>>596071
Не, не смогу, образования недостаточно, только если примерно почувствовать.
- электроны слишком лёгкие?
- даже если виртуальную пару растащить и сделать реальной - она сразу анигилируется об соседей?
- впринципе виртуальную пару ЭМ полем не растащить в две реальные частицы?
- пока будешь растаскивать и перенаправлять - результирующий импуль погасится в ноль?
я хз короче.
Аноним 18/08/20 Втр 23:34:33 596074522
>>596041
Кстати, а может кто считал. Может даже с учётом того что нужны будут тонны гелия для охлаждения, эти тонны компенсируются за счёт миллионов кельвинов? При забывании того, что это всё стоит охринилиард. Или всякие ЯРД эффективнее?
Аноним 18/08/20 Втр 23:37:18 596076523
Кстати про прямоточность. Почему нельзя сделать регулируемый угол атаки лопастей? Или это не влияет?
Аноним 19/08/20 Срд 00:27:22 596079524
unnamed.jpg 31Кб, 512x344
512x344
Аноним 19/08/20 Срд 04:11:54 596107525
>>596073
Давай подскажу. Почему ЧД не движется никуда от Хокинговского излучения?
Настройки X
Ответить в тред X
15000
Макс объем: 40Mб, макс кол-во файлов: 4
Кликни/брось файл/ctrl-v
Стикеры X
Избранное / Топ тредов